You are on page 1of 98

Otorhinolaryngology - Exam Tests

Соотношение 30%-40%-30%
На запоминание -105, понимание – 140 , применение – 105.

1. Indicate where is the opening of the frontal sinuses?


A. in the upper nasal passage
B. in the middle nasal passage
C. in the lower nasal passage
D. in the common nasal passage
E. into the nasopharynx

2. Indicate in which nasal passage the nasolacrimal canal opens?


A. to the top
B. in the inferior
C. in the middle
D. into the nasopharynx
E. in general

3. Indicate which of the cartilage is the skeleton of the nasal septum?


A. pterygoid
B. quadrangular
C. sesamoid
D. triangular
E. additional

4. Indicate where is the opening of the maxillary sinuses?


A. upper nasal passage
B. lower nasal passage
C. middle nasal passage
D. lateral wall of the nasopharynx
E. medial wall of the nasal cavity

5. Indicate where is the opening of the anterior and middle cells of the
ethmoid labyrinth?
A. upper nasal passage
B. nasal septum
C. lower nasal passage
D. middle nasal passage
E. lateral wall of the nasopharynx

6. Indicate where is the openingof the posterior cells of the ethmoid labyrinth?
A. upper nasal passage
B. common nasal passage
C. lower nasal passage
D. middle nasal passage
E. lateral wall of the nasopharynx

7. Indicate where is the opening of the sphenoid sinuses?


A. upper nasal passage
B. nasal septum
C. lower nasal passage
D. middle nasal passage
E. lateral wall of the nasopharynx

8. Froom what cells consist of the epithelial cover of the upper nasal passage?
A. olfactory
B. goblet
C. supporters
D. secretory
E. serous
9. Indicate which artery of the nasal cavity is the main one?
A. internal carotid artery
B. maxillary artery
C. wedge-palatine artery
D. anterior ethmoid artery
E. nasopalatine artery

10. Which of the named nerves carry out the sensory innervation of the nasal
cavity?
A. olfactory nerve
B. third branch the trigeminal nerve
C. second branch of the trigeminal nerve
D. chorda thympani
E. glossopharyngeal nerve

11. Indicate which of the listed arteries provide blood supply of the frontal
sinus?
A. maxillary, facial, ocular
B. maxillary, ascending pharyngeal, ocular
C. anterior ethmoid, infraorbital, middle cerebral
D. ocular, maxillary, superficial temporal
E. facial, posterior ethmoid, ascending pharyngeal

12. Indicate the anatomical structures that are visible on posterior


rhinoscopy?
A. nasal septum, middle and inferior turbinates
B. posterior pharyngeal wall, superior and middle turbinates
C. common nasal passage, middle and lower nasal passage
D. upper pole of palatine tonsils, tubal tonsils
E.nasopharynx, choanae, posterior ends of turbinates
13.Which of the following sinuses open in the anterior-superior hiatus
semilunaris?
A. frontal
B. ethmoidal
C. maxillary
D. wedge
E. maxillary

14. On which of the walls of the maxillary sinus is the opening connecting it
with the nasal cavity?
A. medial
B. top
C. lower
D. back
E. anterior

15. Which of the walls of the maxillary sinus limits it from the pterygopalatine
fossa?
A. medial
B. top
C. lower
D. back
E. anterior

16. On which place of the sinus frontales is the opening of the frontal-nasal
canal?
A. front
B. back
C. top
D. bottom
E. lateral

17. Where is in the nasal cavity located is the regio respiratoria?


A. from the bottom of the nasal cavity to the upper edge of the inferior
turbinate
B. from the bottom of the nasal cavity to the lower edge of the superior turbinate
C. from the bottom of the nasal cavity to the lower edge of the middle turbinate
D. from the inferior turbinate to the superior turbinate of the nasal cavity
E. from the superior turbinate to the bottom of the nasal cavity

18. Where is in the nasal cavity located is the regio olfactoria?


A. in the area of the bottom of the nasal cavity
B. from the bottom of the nasal cavity to the inferior turbinate
C. from the bottom of the nasal cavity to the lower edge of the middle turbinate
D. from the middle turbinate to the vault of the nasal cavity
E. from the bottom of the nasal cavity to the superior turbinate

19. What formations are involved in the lower wall or floor of the maxillary
sinus?
A. pterygopalatine space
B. lateral wall of the nasal cavity
C. alveolar process of the upper jaw
D. peripharyngeal space
E. m. Masseter

20. Indicate which of the listed sinuses open in the posterior hiatus
semilunaris?
A. frontal
B. anterior and middle cells of the ethmoid labyrinth
C. maxillary
D. wedge
E. posterior cells of the ethmoid labyrinth

21. Where is the nasopharyngeal opening of the Eustachian tube located?


A. on the lateral wall of the nasopharynx
B. in the region of the inferior turbinates
C. in the area of the upper nasal passage
D. under the inferior turbinate
E. under the middle turbinate
22. Point out where the tympanic opening of the Eustachian tube opens?
A. nasopharynx
B. into the oropharynx
C. in the hypotympanum
D. in mesotympanum
E. in epitympanum

23. What bone formations form the lateral skeleton of the nasal cavity?
A. sieve plate of ethmoid bone
B. palatine process of ethmoid bone
C. horizontal plate of the palatine bone
D. perpendicular plate of ethmoid bone
E. frontal process of the upper jaw

24. What anatomical structures make up the upper wall of the nasal cavity?
A. horizontal plate of the palatine bone
B. frontal process of the upper jaw
C. perpendicular plate of the palatine bone
D. sieve plate of the ethmoid bone
E. pterygoid process of the sphenoid bone
25. Indicate where the hemilunar slit of N.I. Pirogov?
A. in the area of the lateral wall of the middle nasal passage
B. in the area of the lateral wall of the upper nasal passage
C. in the area of the lateral wall of the lower auditory canal
D. in the region of the medial wall of the nasal cavity
E. in the area of the bottom of the nasal cavity

26. Where is the bleeding zone or locus Kissilbachii located?


A. in the anteroinferior part of the nasal septum
B. in the posterior part of the nasal septum
C. in the area of the bottom of the nasal cavity
D. in the posterior nasal septum
E. in the area of the lateral wall of the nasal cavity

27. What does the posterior wall of the maxillary sinus border with?
A. with the lower wall of the orbit
B. with the outer wall of the nasal cavity
C. with pterygopalatine fossa
D. fit teeth cells
E. with inferior turbinate

28. What is the normal state of the nasal mucosa?


A. pink, moist, smooth
B. red, moist, bumpy
C. bluish, moist, bumpy
D. bluish color, dry, smooth
E. red, dry, smooth

29. What anatomical structures are located on the lateral wall of the nasal
cavity?
A. nasal bones
B. quadrangular cartilage
C. lacrimal bone
D. turbinates
E. ethmoid sells

30. What anatomical structures make up the nasal septum?


A. frontal process of the maxillary sinus, ethmoid cells, lacrimal bone, lateral
cartilage
B. medial wall of the maxillary sinus, lacrimal bone, ethmoid cells, wing cartilage
C. nasal bones, palatine process of the upper jaw, nasal process of the frontal bone
D. perpendicular plate of ethmoid bone, 4-angular cartilage, vomer
E. nasal bones, ethmoid plate, sphenoid bone

31. How many turbinates are located in each half of the nasal cavity?
A. 1
B. 2
C. 3
D. 4
E. 5

32. Indicate which amygdala is located at the mouth of the Eustachian tubes?
A. palatine
B. pharyngeal
C. tubal
D. lingual
E. laryngeal

33. List which of the presented formulations characterize the pharynx?


A. hollow elastic tube, which is an extension of the laryngopharynx
B. muscular cavity located in front of the cervical vertebrae
C. the muscular tube that connects the hypopharynx to the stomach
D. hollow elastic tube, which is an extension of the trachea
E. fissure located in front of the cervical part of the vertebrae, which is filled with
loose connective tissue

34. Give a definition of what is the pharyngeal space?


A. hollow elastic tube, which is an extension of the larynx
B. cavity with muscular walls, located in front of the cervical vertebrae
C. the muscular tube that connects the hypopharynx to the stomach
D. hollow elastic tube, which is an extension of the trachea
E. narrow fissure located in front of the cervical vertebrae

35. What is the border between the upper and middle parts of the pharynx?
A. choanas, opener, bottom of the nasal cavity
B. posterior ends of the turbinates, vault of the nasopharynx,
C. horizontal level of hard and soft palate
D. plane, mentally drawn from the uvula to the back of the pharynx
E. posterior ends of the middle turbinates, vomer, choanae

36. By what of anatomical formation of the nasopharynx communicates with


the nasal cavity?
A. pharynx
B. choanes
C. soft palate
D. oropharynx
E. pharynx

37. What is the length of the nasal part of the pharynx (nasopharynx,
epipharynx)?
A. to the level of the root of the tongue
B. before entering the esophagus
C. to the level of the hard palate
D. before going into the larynx
E. before transition to trachea

38. What is the length of the laryngeal part of the pharynx (hypopharynx,
hypopharynx)?
A. to the level of the root of the tongue
B. before entering the esophagus
C. to the level of the hard palate
D. before going into the larynx
E. before transition to trachea

39. Determine how many choanes open into the nasopharynx?


A. 1
B.2
C.3
D.4
E.5

40.Indicate which anatomical structures do the lateral walls of the pharynx


border with?
A. choanami and opener
B. processes of the cervical vertebrae
C. neurovascular bundle of the neck
D. pharynx and posterior arches of the tonsils
E. the anterior wall of the sphenoid sinus

41. Choose the correct answer: the pharynx starts from the base of the skull
and reaches the level?
A. 1 thoracic vertebra
B. 4 cervical vertebrae
C. 5 cervical vertebra
D. 6 cervical vertebra
E. 7 thoracic vertebra

42.Indicate which mentally drawn plane is the border of the middle and lower
parts of the pharynx?
A. through the soft palate to the spine
B. through the hard palate to the spine
C. from the upper pole of the tonsils to the spine
D. from the upper third of the tonsils to the spine
E. from the root of the tongue to the spine

43. Indicate which of the tonsills begins to form earlier than the others?
A. lingual
B. pharyngeal
C. pipe
D. side bolsters
E. palatine

44. Indicate which muscle expands the glottis?


A. thyroid-hyoid
B. sterno-thyroid
C. sternohyoid
D. lateral ring-arytenoid
E. posterior ring-arytenoid
45. Indicate which muscle plays the main role in the process of voice
formation?
A. internal thyroid
B. scary laryngeal
C. thyroid hyoid
D. cricothyroid
E. shield-epiglottis

46. Indicate which of the listed functions is characteristic of the larynx?


A. digestive
B. ventilation
C. drainage
D. resonator
E. dividing

47. How many parts of the larynx do you know?


A. 1
B. 2
C. 3
D. 4
E. 5

48. List, which you know all the unpaired cartilages of the larynx?
A. epiglottis, thyroid, cricoid
B. arytenoid, horn-shaped, wedge-shaped
C. epiglottis, thyroid, arytenoid
D. wedge-shaped, thyroid, arytenoid
E. wedge-shaped, thyroid, cricoid

49. What are the elastic cartilages of the larynx?


A. epiglottis, thyroid, cricoid
B. arytenoid, sontorinia, vrisbergiev
C. thyroid, cricoid, arytenoid
D. arytenoid, horn-shaped, wedge-shaped
E. epiglottis, sontorinii, vrisbergiev

50. Determine which epithelium covers the vocal cords, the parietal space, the
tubercle and the lingual surface of the epiglottis?
A. cylindrical, ciliated
B. goblet
C. multilayer flat
D. Deiters cells
E. cubic
51. What cartilage is the basis of the skeleton of the larynx?
A. thyroid
B. arytenoid
C. cricoid
D. horn-shaped
E. epiglottis

52. List which cartilages of the larynx are hyaline?


A. thyroid, wedge-shaped, horn-shaped
B. cricoid, arytenoid, wedge-shaped
C. epiglottis, horn-shaped, thyroid
D. thyroid, cricoid, arytenoid
E. epiglottis, cricoid, thyroid

53. How many groups are the internal muscles of the larynx divided into?
A. 2
B. 3
C. 4
D. 5
E. 6

54. What is the function of the posterior cricoid muscle of the larynx?
A. widens the glottis
B. narrows the glottis
C. brings the arytenoid cartilage closer to each other
D. tightens the vocal cords
E. lowers the epiglottis end
55. What is the function of the scapular laryngeal muscles of the larynx?
A. widens the glottis
B. narrows the glottis
C. lowers the epiglottis
D. tightens the vocal cords
E. brings the arytenoid cartilage closer to each other

56. What is the function of the lateral cricoid muscle of the larynx?
A. narrows the glottis
B. lowers the epiglottis
C. tightens the vocal folds
D. brings the arytenoid cartilage closer to each other
E. dilates the glottis

57. What is the function of the oblique and transverse arytenoid muscles?
A. narrows the glottis
B. tightens the vocal cords
C. brings the arytenoid cartilage closer to each other
D. expands the glottis
E. lowers the epiglottis

58. What is the function of the anterior cricoid-thyroid muscle of the larynx?
A. lowers the epiglottis
B. strains the vocal cords
C. widens the glottis
D. tightens the vocal cords
E. narrows the glottis

59. What is the function of m.vocalis?


A. lowers the epiglottis
B. strains the vocal cords
C. dilates the glottis
D. tightens the vocal cords
E. narrows the glottis

60. What is the reason for the symptom "crunch of the larynx"?
A. rubbing the arytenoid cartilage with each other
B. rubbing arytenoid cartilage with cricoid cartilage
C. rubbing the thyroid cartilage with cricoid cartilage
D. rubbing the thyroid cartilage with the epiglottis
E. friction of arytenoid cartilage with accessory cartilage

61. What cartilage is the foundation of the larynx?


A. thyroid
B. arytenoid
C. cricoid
D. horn-shaped
E. epiglottis
62. How many cartilaginous semirings does the "skeleton" of the trachea
consist of?
A. 8-10
B. 10-12
C. 16-20
D. 23-25
E. 26-30

63. What is the membranous labirinth filled with?


A. tissue fluid
B. blood
C. endolymph
D. cerebral fluid
E. perilymph

64. What function of the ear is impaired in tympanic otosclerosis?


A. combined damage to air and bone conduction
B. ventilation
C. perception of sounds through the bone
D. aerial sound conduction
E. barometric

65. What function of the ear is impaired in mixed otosclerosis?


A. air and bone conduction
B. ventilation and drainage conductivity
C. bone and ventilation
D. air and drainage conductivity
E. barometric and arterial conduction

66. What function of the ear is impaired in cochlear otosclerosis?


A. combined damage to air and bone conduction
B. ventilation
C. perception of sounds through the bone
D. aerial sound conduction
E. barometric
67. What anatomical formation is responsible for sound perception?
A. mastoid process
B. tympanic membrane
C. organ of Corti
D. auditory ossicles
E. external auditory canal

68. With what is the bone labyrinth communicated through the window of the
vestibule?
A. with an auditory tube
B. with the anterior cranial fossa
C. with a tympanic cavity
D. mastoid
E. with posterior cranial fossa

69. Where is the spiral organ located?


A. in the vestibule
B. in the membranous cochlea
C. on the mastoid
D. in the tympanic cavity
E. in semicircular canals

70. Indicate what is the membranous channel of the labyrinth filled with?
A. tissue fluid
B. CSF
C. endolymph
D. cerebral fluid
E. perilymph

71. The tympanic cavity communicates with the bone labyrinth by means of?
A. eustachian tube
B. plumbing vestibule
C. Snail water pipe
D. oval and round windows
E. external auditory canal

72. What is the use of quantitative hearing research?


A. catheter
B. frontal reflector
C. pneumatic Siegle funnel
D. audiometer
E. tuning fork

73. Indicate at what distance from the ear is perceived whisper speech
normally?
A. 0.5 m
B. 2 m
C. 6 m
D. 8 m
E. 10 m

74. What does the anterior wall of the tympanic cavity border with?
A. mastoid, antrum
B. bulb of the jugular vein
C. internal carotid artery
D. tympanic membrane
E. middle cranial fossa, temporal lobe of the brain

75. What does the posterior wall of the tympanic cavity border with?
A. mastoid
B. bulb of the jugular vein
C. internal carotid artery
D. tympanic membrane
E. middle cranial fossa, temporal lobe of the brain

76. What does the upper wall of the tympanic cavity border with?
A. mastoid, antrum
B. bulb of the jugular vein
C. internal carotid artery
D. tympanic membrane
E. middle cranial fossa

77. What does the lateral wall of the tympanic cavity border with?
A. mastoid
B. bulb of the jugular vein
C. internal carotid artery
D. tympanic membrane
E. middle cranial fossa

78. What does the lower wall of the tympanic cavity border with?
A. mastoid, antrum
B. bulb of the jugular vein
C. internal carotid artery
D. tympanic membrane
E. middle cranial fossa
79. What does the frontal wall of the external auditory canal border with?
A. mastoid
B. joint of the lower jaw
C. parotid gland
D. middle cranial fossa
E. internal carotid artery

80. What does the posterior wall of the external auditory canal border with?
A. mastoid
B. articular capsule of the lower jaw
C. parotid gland
D. middle cranial fossa
E. internal carotid artery

81. What does the upper wall of the external auditory canal border with?
A. mastoid
B. articular capsule of the lower jaw
C. parotid gland
D. middle cranial fossa
E. internal carotid artery

82. What does the lower wall of the external auditory canal border with?
A. mastoid
B. articular capsule of the lower jaw
C. parotid gland
D. middle cranial fossa
E. internal carotid artery

83. What does the medial wall of the tympanic cavity border with?
A. mastoid
B. articular capsule of the lower jaw
C. outer wall of the labyrinth
D. middle cranial fossa
E. internal carotid artery

84. What departments does the organ of hearing consist of?


A. from the outer, middle and inner ear
B. from the auricle, the ear canal of the tympanic cavity
C. pathways of the brain and auditory cortex
D. Corti's organ of the pathways of the brain
E. bone and membranous labyrinth

85. List the anatomical departments of the sound-conducting part of the


auditory analyzer?
A. organ Corti , pathways and cerebral cortex
B. semicircular canals and the vestibule of the labyrinth
C. ampullary receptor apparatus, liquid media
D. otolith receptor apparatus, liquid media
E. ear canal, timpanic cavity, fluids

86. Indicate on which part of the tympanic cavity the round window is
projected?
A. on the antrum
B. eustachian tube
C. mastoid cells
D. on the hypotympanum area
E. on the eardrum

87. Indicate the correct answer: in the normal state, the lumen of the auditory
tube:
A. gapes constantly during physical activity
B. close when talking and shouting
C. partially open during takeoff and landing by aircraft
D. partially closed during rest and sleep
E. opens on swallowing and chewing

88. What anatomical formation is responsible for sound perception?


A. mastoid process
B. tympanic membrane
C. organ Corti
D. auditory ossicles
E. external auditory canal

89. Where is the spiral organ located?


A. in the cochlea
B. in the membranous cochlea
C. on the eve
D. in the tympanic cavity
E. in semicircular canals

90. Indicate in what range of sound perception is human hearing normal?


A. electric current from 40 to 70 thousand oscillations per second
B. periodic pressure increases from 21 to 30 thousand oscillations per second
C. ultrasounds - sound from 30 thousand to 40 thousand vibrations per second
D. infrasounds - sound from 5 to 16 vibrations per second
E. sound from 16 to 20 thousand vibrations per second

91. How many departments does the auditory analyzer have?


A. 1
B. 2
C. 3
D. 4
E. 5

92. What anatomical formation is a kind of horn that collects and directs
sound waves?
A. tympanic cavity
B. turbinate
C. auricle
D. eustachian tube
E. ear canal

93. What anatomical structure is used to conduct sound vibrations into the
middle ear and selectively amplify them at separate (resonant) frequencies?
A. internal auditory canal
B. external auditory canal
C. narrowing of the external auditory canal
D. vestibule of the external auditory canal
E. cochlear passage

94. What anatomical formation belongs to the middle ear?


A. first curl of the snail
B. tympanic cavity
C. semicircular canals
D. drum ladder
E. front staircase

95. Resorption of what tissue in the middle ear of a newborn is accompanied


by the formation of air cells in the temporal bone?
A. muscle tissue
B. epidermal tissue
C. myxoid tissue
D. mucous membrane
E. mature connective tissue

96. What part of the auricle is devoid of a cartilaginous base?


A. tragus
B. antitragus
C. antihelix
D. lobe
E. curl
97. Which of the named anatomical structures communicate with the upper
nasal passage?
A. nasolacrimal canal
B. sphenoid sinus
C. maxillary sinus
D. anterior cells of the ethmoid labyrinth
E. frontal sinus

98. What is the function of the auditory tube?


A. respiratory
B. drainage
C. vocal
D. phonator
E. swallowing

99. Indicate where is the nasopharyngeal opening of the auditory tube?


A. in the area of the fornix of the nasopharynx
B. on the lateral wall of the nasopharynx
C. in the area of the upper nasal passage
D. under the inferior turbinate
E. under the middle turbinate

100. Indicate where the tympanic opening of the auditory tube opens?
A. nasopharynx
B. into the oropharynx
C. in the hypotympanum
D. in mesotympanum
E. in epitympanum

101. Functions of the nasopharynx?


A. respiratory
B. digestive
C. olfactory
D. protective
E. drainage

102. Digital examination of the nasopharynx is performed at?


A. fracture of the nose
B. acute rhinitis
C. adenoid vegetation
D. chronic rhinitis
E. eustacheitis

103. What is examined during posterior rhinoscopy?


A. oropharynx
B. nasopharynx
C. laryngopharynx
D. nasal cavity
E. sphenoid sinuses

104. In which disease, of the listed, develops sensorineural hearing loss?


A. acute otitis media
B. mesotympanitis
C. epitympanitis
D. cochlear neuritis
E. middle ear catarrh
105. What is the basis of middle ear catarrh?
A. development of sensorineural hearing loss
B. exposure to ototoxic drugs
C. trauma
D. dysfunction of the auditory tube
E. manifestation of endolymphatic hydrops

106. What is the method of surgical intervention for hypertrophic rhinitis?

A. galvanocaustics

B. excision of synechiae of the nasal cavity

C. adenotomy
D. conchotomy

E. cryotherapy

107. Which of the following methods of treatment of otogenic


intracranial complications are the main one?
A. use of anticoagulants
B. antibiotic therapy
C. pathogenetic therapy
D. symptomatic therapy
E. surgical intervention

108. What method is most informative in the diagnosis of brain abscesses:


A. x-ray of the skull
B. angiography of the brain
C. echoencephalography
D. computed tomography of the brain
E. electroencephalography

Comprehension tests.

1. A 30-year-old patient came to the emergency room with complaints of tinnitus


and hearing loss on both sides. From the anamnesis: the above complaints
appeared after the flu two weeks ago. Otoscopic picture is normal. The auditory
passport indicates a bilateral hearing loss of the type of violation of the sound
perception system. What is the most likely diagnosis?

A. otosclerosis
B. exudative otitis media
C. cochlear neuritis
D. Meniere's disease
E. acute mastoiditis
2. A 28-year-old patient consulted an ENT doctor with complaints of pain in the
right ear for 3 days. A few days later, a swelling appeared in the corner of the
lower jaw. An otoscopy revealed signs of a furuncle on the lower wall of the ear
canal. Guess what complication has arisen given the topography?

A. eustachitis
B. cochleoneuritis
C. meningitis
D. mumps
E. mastoiditis

3. The patient, in the absence of changes during otoscopy, has complaints of ear
congestion, hearing loss. What is the most likely diagnosis?

A. exudative otitis media


B. mumps
C. labyrinthitis
D. tubo otitis
E. adhesive otitis media

4. A patient 40 years old, complains of hearing loss in both ears, tinnitus. The
disease developed gradually over the course of 3 years, notes an improvement in
hearing in a noisy environment. Otoscopy is normal. Rinne and Zhelle's
experiences are negative. What is the most likely diagnosis?

A. bilateral cochleoneuritis
B. otosclerosis
C. Meniere's disease
D. middle ear catarrh
E. eustachitis

5. A patient 20 years old, delivered by an ambulance with complaints of severe


pain in the throat on the left, aggravated by movement of the head and neck,
general weakness, malaise. Ob-but: the general condition is severe, the body
temperature is 39 ° C, on palpation the submandibular lymph nodes are enlarged,
painful. Pharyngoscopic: the mouth opens with difficulty due to the trismus of the
masticatory muscles, there is hyperemia, swelling and a spherical protrusion of the
lateral pharyngeal wall on the left along with the amygdala on the affected side.
Provide the most likely diagnosis.

A. phlegmon of the floor of the mouth (Ludwig's angina)


B. phlegmonous tonsillitis
C. monocytic tonsillitis
D. angina Simanovsky-Vincent
E. agranulocytic angina

6. A 45-year-old patient turned to a polyclinic with complaints of frequent


tonsillitis, occurring with high fever, weakness, recurrent pain in the joints, heart.
Regional lymph nodes are enlarged. Pharyngoscopic - enlargement and loosening
of the palatine tonsils, positive symptoms of Giese, Zak, Preobrazhensky,
expansion of the lacunae, in them purulent plugs, tonsils are soldered to the arches
with scars. What is the most likely diagnosis?

A. chronic catarrhal pharyngitis


B. chronic simple tonsillitis
C. chronic compensated tonsillitis
D. chronic decompensated tonsillitis
E. chronic hypertrophic pharyngitis

7. A patient 18 years old, complains of difficulty in nasal breathing, intermittent


nosebleeds, nasal symptoms for 2 years. With anterior rhinoscopy, in the nasal
passages, a purulent discharge. In posterior rhinoscopy, a tumor-like formation of a
reddish color on a broad base with a smooth, even surface, easily humming when
probing. Provide the most likely diagnosis.

A. angiofibroma
B. melanoma
C. mixed tumor
D. hairy polyp
E. hemangioma

8. A 30-year-old female patient complains of intermittent difficulty in nasal


breathing, sneezing, runny nose, itching in the nose. With anterior rhinoscopy: the
mucous membrane of the nasal cavity is swollen, pale, conchas are congested, the
nasal passages are narrowed, there is a mucous-watery discharge, poor contractility
when lubricated with adrenaline. In peripheral blood: eosinophils - 10%;
lymphocytes - 42%. Which of the following diagnoses is most likely?

A. chronic catarrhal rhinitis


B. chronic hypertrophic rhinitis
C. chronic atrophic rhinitis
D. vasomotor rhinitis
E. allergic rhinitis

9. A 3-year-old child, accompanied by his mother, turned to the children's clinic


with complaints of a foreign body in his nose. According to the mother, the day
before, the child put a rounded object from a children's designer into his nose.
Which of the following is most effective for removing a foreign body from the
nasal cavity?

A. nasal tweezers
B. nasal loop
C. nose hook
D. bulbous probe
E. Kulikovsky needle

10. A 35-year-old patient complained of a headache, aggravated by tilting the head,


purulent discharge from the nose for 3 weeks. Objectively: pain on palpation of the
canine fossa, as well as on percussion of the teeth, swelling of the cheek on the
right. With rhinoscopy - swelling of the mucous membrane, purulent discharge in
the middle nasal passage on the right. On the R-graph of the paranasal sinuses, an
intense decrease in the transparency of the maxillary sinus on the right is noted. R-
graphy of the alveolar bay - inflammation of the alveolar ridge 6 in the region of
the bottom of the maxillary sinus on the right. What is the correct diagnosis?

A. acute purulent sinusitis


B. phlegmon of the buccal region
C. odontogenic sinusitis
D. osteomyelitis of the upper jaw
E. maxillary sinusitis

11. A 30-year-old patient complained of hearing loss and noise in the left ear, as
well as periodic mucopurulent discharge from it. Suffering from suppuration from
the ear for many years. At first, the discharge from the ear was constant, then it
became intermittent and the hearing did not improve. Objectively: the general
condition is satisfactory. Otoscopy: A / S - persistent defect in the stretched part -
pars tensa of the tympanic membrane. Tuning-fork studies: on the left, Rinne's
experience is negative, sound lateralization in the left ear, Schwabach's experience
is lengthened. Provide the most likely diagnosis.

A. adhesive otitis media


B. chronic epitympanitis
C. exudative otitis media
D. chronic mesotympanitis
E. chronic epimesotympanitis

12. Choose a method for stopping parenchymal bleeding after bilateral


tonsillectomy surgery:
A. application of a hemostatic forceps,
B. introduction of a hemostatic tampon, with suturing of the arches;
C. infiltration of the tissues of the tonsillar niche with a solution of novocaine
D. imposition of Mikulich forceps and hemostatic sponge
E. hemostatic therapy with a solution of dicinone 250 mg topically

13. The child has complaints of nasal breathing difficulties, frequent colds and
headaches. According to the mother, he breathes at night with his mouth open, on
examination - the face is puffy, drooping lower jaw, malocclusion of the teeth,
high "Gothic" palate, nasal breathing is weakened, with paluyevy examination
Select the method of surgical treatment for hypertrophy of the pharyngeal tonsil III
degree:

A. adenotomy
B. tonsillotomy
C. tonsillectomy
D. removal of the tonsil cyst
E. uvulatomy

14. A patient 43 years old, complained of hearing loss, noise in the right ear after
a runny nose. During otoscopy, the tympanic membrane is retracted, gray in color,
the handle of the hammer is shortened along the way it is determined by the
injection of the vessels. The mucous membrane of the nasal cavity is hyperemic,
edematous, the inferior turbinates are hypertrophied. Whispering speech to the
right ear 2 meters. Rinne's experience is negative, lateralization of sounds in the
sore ear. Make a proper diagnosis:

A. Acute otitis media


B. Acute tubotympanitis
C. Adhesive otitis media
D. Acute sensorineural hearing loss
E. External limited otitis media

15. A 28-year-old patient, against the background of acute otitis media on the
right, developed dizziness with a sensation of counterclockwise movement of
objects, nausea and a single vomiting. Spontaneous nystagmus to the right was
determined. In the Romberg position, the patient deviates to the left. When
performing finger - nasal and finger - finger tests, an overshot to the left was
revealed. Examination of the ear revealed a purulent discharge in the ear canal, a
defect in the tympanic membrane in the mesotympanum, and a pulsating reflex.
When examining hearing, a complete shutdown of the auditory function on the
right was found. Predict what complication of acute purulent otitis media is
observed in the patient?

A. Meniere's disease
B. otosclerosis
C. meningitis
D. labyrinthitis
E. mastoiditis

16. A patient of the 46 years old, taken to the clinic, complaining of dizziness,
single vomiting, spontaneous nystagmus and profuse sweat appeared. He suffers
from suppuration from the ear since childhood. During the pressor test, nystagmus
and dizziness appeared. Otoscopy on the left: a total defect of the tympanic
membrane, pus in the tympanic cavity, the medial wall is infiltrated, edematous,
with areas of hyperplasia. Estimate what complication of chronic suppurative otitis
media is observed in this patient?

A. meningitis
B. brain abscess
C. otitis externa
D. labyrinthitis
E. mastoiditis

17. A 12-year-old child complains of suppuration from the ear of pain and swelling
in the right ear region. A history of suffering for about three weeks. Despite the
ongoing treatment, suppuration persisted, but the pain was relieved. On
examination, in the right ear canal there is abundant purulent discharge, perforation
in the mesotympanum is revealed, the overhang of the posterior upper wall of the
external auditory canal in its bony section is noted. When percussion of the ear
region on the right, pain is noted. Provide the most likely diagnosis.
A. Acute suppurative otitis media, complicated by meningitis
B. Chronic purulent epimesotympanitis complicated by neuritis
C. Acute suppurative otitis media, complicated by mastoiditis
D. Treatment of chronic mesotympanitis
E. Acute otitis externa complicated by acute cochleoneuritis

18. A 42-year-old patient came to the clinic with complaints of shortness of nasal
breathing, headache. She had a history of nasal polypotomy surgery at the place of
residence. Nasal breathing was restored, but after three months the polyps recurred.
X-ray and CT scan of the paranasal sinuses revealed darkening of almost all
paranasal sinuses. The nasal cavity is obturated with polyps. Nasal breathing is
practically absent.
Establish the correct diagnosis.
A. Polypoid etmoiditis
B. Hyperplastic pansinitis
C. Aspirin triad
D. Polyposis rhinosinuitis
E. Allergic rhinitis

19. A 14-year-old patient complained of persistent nasal congestion, difficulty in


nasal breathing. Periodically, the patient has spontaneous nosebleeds, the cause of
which the patient cannot indicate. On examination, moderately edematous nasal
mucosa was revealed.
with a cyanotic shade, without pathological discharge. Examination of the
nasopharynx revealed a tuberous formation covering the choanas. Education has a
dark purple color. Provide the most likely diagnosis.

A. Angiofibroma of the nasopharynx.


B. Chemodectoma
C. Adenoid vegetation
D. Hemangioma
E. Choanal polyp

20. A patient of 45 years old, came to the clinic with complaints of hoarseness,
soreness, rawness and dryness in the throat, cough. The onset of the disease is
associated with the intake of cold beer six days ago. According to the patient, the
disease began with the appearance of a dry cough, then a cough with phlegm. I did
not feel well, I constantly felt discomfort when swallowing. Laryngoscopy
revealed hyperemia of the laryngeal mucosa, more pronounced in the area of the
vocal folds, with areas of punctate hemorrhages. There is a small amount of
viscous phlegm in the lumen of the larynx. Which of the following diagnoses is the
most likely?

A. Acute catarrhal laryngitis.


B. Syphilis of the larynx (erythema stage)
C. Tuberculosis of the larynx (initial form)
D. Laryngeal angina
E. Edematous-infiltrative laryngitis
21. A 25-year-old female patient was admitted to the clinic with complaints of
sharp pain when swallowing, choking on liquid food, fever up to 38 ° C.
According to the patient, she fell ill acutely after acute respiratory infections. With
indirect laryngoscopy - the mucous membrane of the larynx is sharply hyperemic,
edematous, infiltrated in the region of the scapular laryngeal folds and the
epiglottis. Due to infiltration, the epiglottis is limited in mobility. The vocal folds
are thickened, infiltrated, limited in mobility during phonation. Revealed swelling
of regional lymph nodes. Which of the following diagnoses is the most likely?

A. Phlegmonous laryngitis
B. Abscess of the epiglottis
C. Exacerbation of chronic pachydermic laryngitis
D. Acute catarrhal laryngitis
E. Edematous infiltrative laryngitis
22. 42-year-old patient was admitted to the emergency room by an ambulance
with complaints of pain when swallowing, pain, difficulty eating, choking and
even inability to swallow saliva.
In history, a week ago, he suffered a trauma to the pharynx with a meat bone. Then
there were pains during swallowing, which gradually increased, hoarseness of the
voice joined, shortness of breath appeared, and the temperature rose. With indirect
laryngoscopy, the epiglottis is sharply infiltrated, thickened, practically motionless.
On the lingual surface of the epiglottis there is an area of pronounced protrusion, in
the center of which pus is visible. Provide the most likely diagnosis.

A. Phlegmonous laryngitis
B. Laryngeal angina
C. Abscess of the epiglottis
D. Chondroperechondritis of the larynx
E. Edematous infiltrative laryngitis

23. 7-year-old child, acutely ill, after suffering an acute respiratory viral infection.
Suddenly in the middle of the night, I woke up with a paroxysmal barking cough.
The child is restless, tossing about in bed. Breathing is wheezing, sharply difficult,
inspiratory dyspnea is expressed. The retraction of the soft tissues of the jugular
fossa, above and subclavian spaces, and the epigastric region is determined. A
similar state lasted for an hour, after which, profuse sweating appeared, breathing
improved and the child fell asleep. In the morning I woke up almost healthy, but
some hoarseness remained. Which of the following is most likely to confirm the
diagnosis?

A. Diphtheria stenosis of the larynx, true croup


B. Stenosis of the larynx of allergic genesis
C. Sublining laryngitis, false croup
D. Chronic stenosis of the larynx
E. Edematous-infiltrative laryngitis

24. 35-year-old patient was admitted to the ENT clinic with complaints of: severe
dizziness of a systemic nature, nausea, vomiting, ringing in the left ear, balance
disorders. A history of similar seizures was observed in the patient over the past
five years, with each seizure she notes a gradual decrease in hearing. This attack
began suddenly and for no apparent reason. AD and AS without visible pathology.
Which of the following preliminary diagnoses is the most likely?
A. purulent labyrinthitis
B. acute otitis media, complicated by serous otitis media
C. Meniere's disease
D. fistula of the labyrinth
E. otosclerosis
25. The patient, on the advice of a neighbor, independently treated abdominal pain
with high doses of gentamicin. A few days later, he noted a hearing loss on the
right. Otoscopically: both eardrums are pearl-gray, well-contoured markings.
Hearing: on the left - the norm, on the right spoken speech - 5m whisper speech -
1m. Which of the following preliminary diagnoses is the most likely?
A. acute neuritis of the auditory nerve
B. chronic neuritis of the auditory nerve
C. otosclerosis
D. meniere's disease
E. acute eustachitis
26. The patient complains of hearing loss in both ears, tinnitus. Has been sick for 3
years. A year ago, after giving birth, tinnitus increased significantly and hearing
decreased. Notes improved hearing in noisy environments. About: AS and AD -
the external auditory canal is wide, there is no sulfur, the eardrum is shiny, the
identification points are pronounced.
Auditory passport
AD AS
0.5 m SH.R. 0.25 m
2.5 m R.R. 1.5 m
15 sec SW 128 10 sec
(N-60 sec)
30 sec ck 128 28 sec
(N-30 sec)
33 sec from 2048 30 sec
(N-35 sec)
-R-
-W-
Which of the following preliminary diagnosis is the most probable?
A. meniere's disease
B. bilateral sensorineural hearing loss
C. gradenigo syndrome
D. otosclerosis
E. cochleoneuritis
27. A patient who had been taking treatment for streptoderma for a long time
independently increased the streptomycin dosage three times. Within 2-3 months,
he noted a decrease in hearing on the right. Otoscopically: both eardrums are pearl-
gray, well-contoured markings. Hearing: on the left - the norm, on the right,
colloquial speech - 5m, whispering speech - 1m. Choose the most likely
provisional diagnosis?
A. acute neuritis of the auditory nerve
B. chronic neuritis of the auditory nerve
C. otosclerosis
D. meniere's disease
E. acute eustachitis
28. What symptoms are characteristic of otogenic paresis of the facial nerve?
A. facial asymmetry, dry eyeball, decreased salivation, local taste disorder
B. ptosis, decreased salivation
C. disorder of taste in the anterior 2/3 of the tongue
D. decreased sense of smell, facial asymmetry
E. desensitization of the facial skin
29. A 30-year-old female patient came to the emergency room with complaints of
tinnitus and hearing loss on both sides. From the anamnesis: after suffering the flu
two weeks ago, the above complaints appeared. Otoscopy is normal. The auditory
passport indicates a bilateral hearing loss of the type of violation of the sound
perception system. Choose the most likely diagnosis?
A. toosclerosis
B. exudative otitis media
C. cochlear neuritis
D. Meniere's disease
E. acute mastoiditis
30. A patient of 40 years old, complains of hearing loss in both ears, tinnitus. The
disease developed gradually over the course of 3 years, notes an improvement in
hearing in a noisy environment. Otoscopy is normal. Rinne and Zhelle's
experiences are negative. Which of the following preliminary diagnoses is the most
likely?
A. bilateral cochleoneuritis
B. otosclerosis
C. meniere's disease
D. middle ear catarrh
E. eustachitis
31. Such a paradoxical symptom as Paracusis Willisii (which consists in improving
hearing acuity when staying in a noisy environment) is pathognamonic for which
of the following diseases?
A. for Meniere's disease
B. middle ear catarrh
C. otosclerosis
D. cochleoneuritis
E. eustacheitis
34. A 35-year-old female patient complained of hearing loss and noise in the right
ear. Previously suffered from suppuration of the right ear. As a result of the
treatment, suppuration from the ear stopped, but the hearing remained impaired,
and the noise in the ear began to disturb. Objectively: the general condition of the
patient is satisfactory, the temperature is normal. Otoscopy: A / D - the tympanic
membrane is retracted, the scar is changed. An audiometric examination revealed
conductive hearing loss on the right. Which disease from the following are we
talking about?
A. chronic dry mesotympanitis
B. exudative otitis media
C. adhesive otitis media
D. meniere's disease
E. sensorineural hearing loss
35. The phenomenon of accelerated increase in loudness (FUNG) is characteristic
for:
A. sensorineural hearing loss
B. retrocochlear hearing loss
C. toosclerosis
D. labyrinthitis
E. Meniere's disease
36. Patient of 30 years old, worried about noise in the right ear and hearing loss.
Patient's auditory passport: right ear: whispering speech - 3 m. Rinne: positive
(small), Weber: lateralized to the left, Schwabach: shortened;
left ear: whisper speech 6 m. Rinne: positive, Schwabach: the same.
What is the nature of hearing loss according to the auditory passport?
A. conductive hearing loss
B. sensorineural hearing loss
C. mixed hearing loss
D. deafness
E. conductive and sensorineural hearing loss
37. A patient with inflammatory lesions of the inner ear developed symptoms of
intracranial complications. From which cranial fossa we should predict it, taking
into account the anatomical preformed connections of the inner ear?
A. sides of the anterior cranial fossa
B. sides of the posterior fossa
C. sides of the middle cranial fossa
D. sides of the anterior and posterior cranial fossa
E. sides of the middle and anterior cranial fossa
38.Which diffuse purulent labyrinthitis and thrombosis of the sigmoid sinus, an
abscess often occurs:
A. cerebellum
B. temporal lobe of the brain
C. parietal lobe of the brain
D. brain stem
E. occipital lobe of the brain
39. Patient K., 60 years old, who has been suffering from right-sided epitympanitis
for many years, after suffering from acute respiratory viral infections developed a
sharp diffuse headache, nausea and vomiting, not associated with food intake. The
body temperature is increased to 39 ° C., the stiffness of the muscles of the occiput
is determined, a positive Kernig symptom. What complication most likely
developed in this patient?
A. purulent labyrinthitis
B. brain abscess
C. purulent meningitis
D. thrombosis of the sigmoid sinus
E. sepsis
40.  A 9-year-old patient complains of a sore throat more on the right.  Ill for 5
days. Was diagnosed with angina, was carried out treatment. The condition is
serious.  Body temperature 38.6.  The skin is pale. Pharyngoscopy: trismus,
displacement of the right tonsil to the midline and anteriorly. Hyperemia and
infiltration of the anterior arch and soft palate on the right. Enlarged and painful
submandibular  lymph nodes. Which of the following preliminary diagnoses
is most likely?
 
A. acute tonsillitis
B. paratonsillar abscess
C. retropharyngeal abscess
D. rear peritonsillar abscess
E. parapharengial abscess
 
41. A 45-year-old woman was taking antibiotics for bronchopneumonia. She went
to the doctor with complaints of discomfort when swallowing and moderate
pain. The patient's state of health is satisfactory, the temperature reaction is within
normal limits. With pharyngoscopy, the mucous membrane is moderately
hyperemic, the tonsils are hyperplastic, and plaques are determined on their
surface. The raids of an insular character, white-gray in color, rise above the
surface, in places the raids extend beyond the arches.
Which of the following preliminary diagnoses is most likely ?

 A. pharyngeal diphtheria


B. lacunar angina
C. candidiasis of the pharynx
D. aggravation chronic pharyngitis
E. acute pharyngitis

42. A 48-year-old patient complained of a feeling of lack of dexterity when


swallowing, a significant increase in the size of the uvula, soft palate, and
difficulty in breathing. According to the patient, these signs appeared 1.5 hours
ago, after drinking tea with honey. The phenomena grew rapidly. Objectively: the
general condition is satisfactory, but the patient is agitated and worried about his
condition. Breathing is quickened. Pharyngoscopic picture: mucous membrane
pale, edematous, clean. Which of the following are the most
likely preliminary diagnoses?

 A. allergic edema of the mucous membrane throats


B. acute tonsillitis
C. acute pharyngitis
D. the faringomikoza
E. false croup

43. A 19-year-old patient complained of frequent angina, rapid fatigue, and


weakness. Anginas are difficult. The temperature during angina reaches 38.7-39
degrees. Pain in the pharynx appears, especially when swallowing. The patient
goes to bed, weakness, malaise, pain in bones and joints appear. Ill since the age of
16. The general condition is satisfactory. Pharyngscopy: marked persistent
hyperemia edges palatine arches, caseous-purulent tube in the gaps tonsils, tonsils
friable. Which of the following are the most likely preliminary diagnoses?
 
 A. chronic tonsillitis. Toxic-allergic reaction form
B. chronic tonsillitis. Simple form
C. chronic pharyngitis
D. acute tonsillitis
E. paratonsillar abscess

44. A patient is 24 years old, complaints for severe pain in the throat, temperature
38 0 C, weakness, lethargy. Ill for 2 days, after hypothermia.  General condition of
moderate severity, t 38.10 C, pale skin, enlarged regional lymph nodes of the
pharynx are palpable. Pharyngscopy determined hyperemia and infiltration of the
soft palate, palatine arches, edematous and hyperemic tonsils, on the surface of
which there are numerous round, slightly elevated yellow dots. Which of the
following preliminary diagnoses is most likely?
 
 A. lacunar angina
B. catarrhal angina
C. follicular angina
D. mixed angina
E. herpetic angina

45. In patient 26 years, complaining of severe pain in the throat when swallowing,
fever, t 39 0 C, weakness, joint pain, putrid breath. Objectively: the general
condition is severe, temperature 39.6 ° C, the skin is grayish, enlarged regional
lymph nodes of the pharynx are palpable. Pharyngoscopy determined hyperemia
and infiltration of the soft palate and palatine arches, increase and hyperemia of the
tonsils, purulent - fibrinous plaque on the surface of the tonsils. The general
analysis of blood revealed a shift of the leukogram to the left. Which of the
following forms of angina may be the most likely preliminary diagnoses?
 
 A. lacunar
B. catarrhal
C. follicular
D. mixed
E. herpetic

46. A 16-year-old patient has complaints of severe sore throat, chills, temperature
39 ° C, weakness, joint pain, headache. Throat hurts 3rd day. Objectively: general
condition of moderate severity, t 38.6 0 С, pale skin, enlarged regional lymph
nodes are palpable. Opening the mouth is free. Pharyngoscopy: the soft palate is
symmetrical, hyperemia and infiltration of the soft palate and arches, enlarged and
hyperemic palatine tonsils, covered with a dirty white coating. In addition, islets of
gray plaque on the mucous membrane of the nasal and laryngopharynx. Which of
the following are the most likely preliminary diagnoses?
 
 A. acute tonsillitis
B. chronic tonsillitis
C. diphtheria throats
D. scarlatina
E. monocytic angina

47. A 15-year-old patient has complaints for severe pain in the throat, chills,
temperature 39 ° C, weakness, joint pain. The throat hurts for 4 days, before that
during the week worried about weakness, lethargy, malaise, subfebrile
temperature. Objectively: general condition of heavy, t 39,6 0 C.
Skin grayish, palpable enlarged cervical, axillary, inguinal lymph nodes; the liver
and spleen are enlarged. Pharyngoscopy determined hyperemia and infiltration of
the soft palate and palatine arches, enlarged and hyperemic palatine tonsils,
covered with fibrinous plaque. The general analysis of blood found in a large
number of atypical mononukle ares. Which of the following are the most
likely preliminary diagnoses?
 
 A. acute tonsillitis
B. chronic tonsillitis
C. diphtheria throats
D. angina when scarlet fever
E. monocytic angina

48 A 26-year-old patient has complaints of hard pain in the throat, more right side,
temperature 38.2 ° C, weakness. The 5th day is ill, he was treated
by gargling. Objectively: the voice is nasal, the skin is pale, t 38.6 0 C, enlarged
submandibular, retromandibular lymph nodes are palpable, sharply painful on the
right. Pharyngoscopy: the mouth opens wide on one finger, asymmetry hyperemia
of the soft palate, due to infiltration and edema of the right front of the palatine
arches, tonsils enlarged and hyperemic in the gaps of the tonsils purulent
congestion, the right amygdala is shifted to the center, swelling of the
tongue. Which of the following are the most likely preliminary diagnoses?
 
 A. acute tonsillitis
B. lacunar angina, paratonsillitis on the right
C. diphtheria throats
D. angina at a scarlatina.
E. monocytic angina
 
49. A 7- year-old child was taken to the emergency room by the ambulance team
with complaints of difficulty breathing, hard pain in the throat, temperature 38.2 °
C, weakness. From the words of the parents it is known that the third day
is sick. Objectively: a serious condition, stridor breathing, a hoarse voice, pale
skin, temperature 38.6 ° C, enlarged, sharply painful regional lymph nodes are
palpable, the head is tilted to the left. Pharyngoscopy: asymmetry, hyperemia and
protrusion of the posterior pharyngeal wall, more on the left. When feeling with a
blunt probe, a soft swelling is determined. Which of the following are the most
likely preliminary diagnoses?
 
 A. retropharyngeal abscess
B. paratonsillar abscess
C. parapharyngeal
D. phlegmon of the neck
E. chronic tonsillitis

50. A 19- year-old patient has complaints of frequent tonsillitis, recurrent pain in


the joints, and increased fatigue. From the anamnesis it is known that for 3 years
every six months she has been sick with tonsillitis, a year ago she had
paratonsillitis. Objectively: the general condition is satisfactory, the skin is of
normal color, the lower eyelids are cyanotic, enlarged regional lymph nodes are
palpable, the body temperature is normal. Pharyngoscopy: determined palatine
tonsils closing in the midline, purulent plugs in the lacunae. Which of the
following are the most likely preliminary diagnoses?
 
 
 A. chronic tonsillitis. Simple form
B. paratonsillar abscess
C. retropharyngeal abscess
D. chronic tonsillitis II stage
E. peripharyngeal abscess

51. Patients 7 years of complaints about frequent angina (2-3 times a


year). Objectively: the general condition is satisfactory, the skin is of normal color,
the regional lymph nodes are not palpable. Pharyngoscopy: palatine tonsils behind
the arches, soldered to them, hyperemia of the edges of the palatine arches, roller-
like thickening and swelling of the edges of the anterior and posterior
palatine. When pressed with a spatula on the palatine tonsils, liquid pus is released
from the lacunae. Which of the following are the most
likely preliminary diagnoses?
 
 A. chronic tonsillitis. Simple form
B. paratonsillar abscess
C. retropharyngeal abscess
D. chronic tonsillitis TAF II art.
E. peripharyngeal absces

 
52. 58 years old patient has complaints of dryness and scratch in the throat for a
long time. Objectively: the general condition is satisfactory, the skin is of normal
color, painless regional lymph nodes are palpable. Pharyngoscopy:  the back wall
throat pale pink, dry with lacquer Shine. Which of the following are the most
likely preliminary diagnoses?
 
A. atrophic pharyngitis
B. acute tonsillitis
C. acute pharyngitis
D. the faringomikoza
E. hypertrophic pharyngitis

53. A 67 years old patient has complaints of dryness and scratch in the throat,


sometimes burning and pain when swallowing solid food, and bad breath. Ill for
more than 5 years; in the anamnesis - suffers from chronic achilic gastritis and
cholecystitis. With mesophangoscopy - the mucous membrane of the pharynx is
pale, thinned, dry, injected with vessels, covered with viscous mucus. Which of the
following are the most likely preliminary diagnoses?
 

A. atrophic pharyngitis 
B. acute tonsillitis
C. acute pharyngitis
D. the faringomikoza
E. hypertrophic pharyngitis
 
54. A 45 years old patient has complaints of a feeling of flushing and dryness in
the throat. These complaints have been disturbing for 6-8
months. When mesophangoscopy: mucosa of the posterior wall of the pharynx
hyperemic, thickened injected vessels, covered with thick mucus; single granules
of lymphadenoid tissue are visible . Which of the following are the most
likely preliminary diagnoses?
 
 A. atrophic pharyngitis
B. acute tonsillitis
C. acute pharyngitis
D. the faringomikoza
E. hypertrophic pharyngitis

55.  The patient 19 years of complaints of hard pain in the throat, worse
swallowing, on fever, malaise, weakness, headache. Ill for 3 days, the onset of the
disease is associated with hypothermia. Temperature 38.3 0 С, pulse - 88 bpm,
rhythmic. On palpation, the retromandibular and submandibular lymph nodes
are enlarged and painful. With mesopharyngoscopy - bright hyperemia and
swelling of the mucous membrane of the tonsils, on the surface of which there are
multiple white dots the size of a millet grain. Which of the following forms of
angina are the most likely preliminary diagnoses?
 
 A. lacunar
B. catarrhal
C. follicular
D. mixed
E. herpetic

56. A 48-year-old patient has complaints of hard pain in the throat that prevents
swallowing, fever, aching in the limbs, malaise. The disease began acutely, two
days ago. The patient's condition is of moderate severity, the skin is dry, of
a grayish- yellow hue. Temperature 39 ºС, pulse 88 bpm. The tonsils of the palate
are covered with dirty gray necrotic plaques. Regional lymph nodes are
enlarged. In the study of blood in the formula, pronounced leukopenia is
determined, a significant decrease in the number
of neutrophilic granulocytes. Which of the following are the most
likely preliminary diagnoses?

 A. acute tonsillitis


B. agranulocytic angina
C. diphtheria
D. scarlatina
E. monocytic angina

57. A 6- year-old child often suffers from respiratory diseases, the first time in the


last three years had a sore throat. There is a sharp increase in the palatine tonsils,
which are almost closed along the midline. There are no deposits and plugs in the
tonsils. There are no signs of chronic tonsillitis. Which of the following are
the most likely preliminary diagnoses?
A. follicular angina
B. lacunar angina
C. diphtheria of the pharynx
D. chronic tonsillitis
E. hypertrophy of the tonsils III art.
 
58. During a medical examination, a 35-year-old patient had dense whitish-yellow
"plugs" on the surface of the palatine and lingual tonsils. "Plugs" have a conical
shape, rise above the surface of the mucous membrane, are not removable, of
dense consistency (bone). Which of the following are the most
likely preliminary diagnoses?
 A. leptotrichia throat
B. follicular angina
C. lacunar angina
D. diphtheria of the pharynx
E. ulcerative-necrotic disease angina
 
59. Patient 26 years professional examinations revealed a grayish-yellow coating
on the surface of the right palatine tonsil. After removing the plaque, an ulcer with
uneven edges and a bleeding surface is visible. Body temperature 37.1 C,
enlarged submandibular lymph nodes on the right. Which of the following are
the most likely preliminary diagnoses?
A. ulcerative-necrotic angina
B. follicular angina
C. lacunar angina
D. diphtheria of the pharynx
E. the faringomikosus
60. A 32-year-old patient consulted an ENT doctor with complaints of bursting
pain in the nose and difficulty in nasal breathing. According to the words 3 days
ago, he was hit in the nose. With anterior rhinoscopy, the nasal passages are
sharply narrowed due to inflammatory edema of the mucous membrane in the
anterior sections of the nasal septum on both sides, nasal breathing is difficult.
When probing, there is a symptom of fluctuation, pain. Make a preliminary
diagnosis.

A. curvature of the nasal septum


B. hematoma of the nasal septum
C. abscess of the nasal septum
D. hemangioma of the septum of the nose
E. papilloma of the septum of the nose.

61. A 35-year-old female patient complains of hearing loss and noise in the right
ear. Previously suffered from suppuration of the right ear. As a result of treatment,
suppuration from the ear stopped, but hearing remained impaired, and noise in the
ear began to disturb. Objectively: the general condition of the patient is
satisfactory, the temperature is normal. Otoscopy: A / D - the tympanic membrane
is retracted, the scar is changed. An audiometric examination revealed conductive
hearing loss on the right. What disease are we talking about?

A. chronic dry mesotympanitis


B. exudative otitis media
C. adhesive otitis media
D. Meniere's disease
E. sensorineural hearing loss

62. A 30-year-old patient complained of hearing loss and noise in the left ear, as
well as recurrent mucopurulent discharge from it. Suffering from suppuration from
the ear for many years. At first, the discharge from the ear was constant, then it
became intermittent and the hearing did not improve. Objectively: the general
condition is satisfactory. Otoscopy: A / S - persistent defect in the pars tensa of the
tympanic membrane. Tuning forks: Rinne's experience is negative on the left,
sound lateralization in the left ear, Schwabach's experience is lengthened. Make a
diagnosis.

A. adhesive otitis media


B. chronic epitympanitis
C. exudative otitis media
D. chronic mesotympanitis
E. chronic epimesotympanitis
63. A 40-year-old patient complains of pain in the right ear, inability to open his
mouth, severe pain in the ear when chewing and swelling behind the ear, hearing
loss. Sick for 5 days, at first there was itching in the ear, then pains appeared,
which gradually increased. It became difficult to open the mouth, the temperature
rose, the hearing decreased. Ob-but: the general condition is satisfactory, t - 37.2 °
C, the mouth opens with difficulty, pain when pressing on the tragus and pulling
on the auricle. Otoscopy : A / D - the external auditory canal is sharply narrowed
due to an inflammatory infiltrate on the back wall of it. Make a diagnosis.

A. acute otitis media


B. acute mastoiditis
C. BTE lymphadenitis
D. otitis external
E. chondroperichondritis

64. A 30-year-old patient has turned to an ENT doctor with complaints of tinnitus,
hearing impairment, from the anamnesis: suffers for 2 years, hearing progressively
worsens. Otoscopy is normal. The auditory passport and threshold audiometry
indicate a bilateral hearing loss by the type of damage to the sound-receiving
apparatus. What disease are we talking about?
A. tympanosclerosis
B. exudative otitis media
C. cochleoneuritis
D. Meniere's disease
E. chronic mastoiditis

65. A 42-year-old patient complains of hoarseness, recurrent cough and dry throat
for several months. With indirect laryngoscopy - hyperemia of the mucous
membrane of the larynx, nodules on the vocal folds, areas of pachiderma in the
region of arytenoid cartilage and inter-head space. Make a preliminary diagnosis.

A. atrophic laryngitis
B. catarrhal laryngitis
C. infiltrative laryngitis
D. edematous-polyposis laryngitis
E. hyperplastic laryngitis

66. A 19-year-old patient complains of a headache, aggravated by tilting the head,


purulent nasal discharge for 3 weeks. Objectively: pain on palpation of the canine
fossa, as well as on percussion of the teeth, swelling of the cheek on the right. With
rhinoscopy - swelling of the mucous membrane, purulent discharge in the middle
nasal passage on the right. R-graph of the paranasal sinuses diffuse lowering of the
transparency of the maxillary sinuses on the right. R-graphy of the alveolar bay -
inflammation of the alveolar ridge 6 in the area of the bottom of the maxillary
sinus on the right. Make a preliminary diagnosis?
A. acute purulent sinusitis
B. phlegmon of the buccal region
C. odontogenic sinusitis
D. osteomyelitis of the upper jaw
E. maxillary sinusitis

67. A 45-year-old patient complains of a headache aggravated by tilting the head,


purulent nasal discharge for 2 weeks. Objectively: tenderness on palpation of the
canine fossa on the right. With anterior rhinoscopy - swelling of the nasal mucosa,
purulent discharge in the middle nasal passage on the right. With R-graphy of the
paranasal sinuses, homogeneous darkening of the maxillary sinus on the right is
noted. Make a preliminary diagnosis?

A. acute purulent sphenoiditis


B. acute purulent frontal sinusitis
C. odontogenic sinusitis
D. acute purulent ethmoiditis
E. acute purulent sinusitis

68. What pharyngoscopic picture is typical for catarrhal sore throat?

A. hyperemia and moderate enlargement of the tonsils


B. diffuse hyperemia of the pharyngeal mucosa
C. hyperemia of the mucous membrane of the posterior pharyngeal wall and soft
palate
D. enlargement and hyperemia of the palatine tonsils, on the surface of their
festering follicles in the form of millet grains
E. hyperemia and sharp edema of the mucous membrane of the floor of the mouth

69. What pharyngoscopic picture is typical for follicular tonsillitis?

A. hyperemia and moderate increase in palatine tonsils, purulent deposits on them


B. diffuse hyperemia of the pharyngeal mucosa
C. hyperemia of the mucous membrane of the posterior pharyngeal wall and soft
palate
D. the presence of suppurative follicles in the form of grains on the tonsils
E. hyperemia and sharp edema of the mucous membrane of the floor of the mouth

70. What pharyngoscopic picture is characteristic of a paratonsillar abscess?

A. hyperemia and moderate enlargement of the tonsils


B. diffuse hyperemia of the pharyngeal mucosa
C. hyperemia of the mucous membrane of the posterior pharyngeal wall and soft
palate
D. hyperemia and globular protrusion on the lateral pharyngeal wall
E. hyperemia and sharp edema of the mucous membrane of the floor of the mouth

71. A 30-year-old patient complains of tinnitus, hearing loss. Hearing


progressively worsens. Otoscopy is normal. The auditory passport indicates a
bilateral hearing loss by the type of damage to the sound-receiving apparatus. What
disease are we talking about?

A. otosclerosis
B. exudative otitis media
C. cochlear neuritis
D. Meniere's disease
E. chronic mastoiditis

72. Specify the characteristic signs of vasomotor rhinitis with anterior rhinoscopy:

A. hyperemia and hypertrophy of the turbinates thinning of the mucosa


B. edema and cyanosis of the nasal turbinate mucosa, Voyachek's gray spots
C. hyperemia and swelling of the mucous membrane, narrowing of the nasal
passages
D. swelling of the mucous membrane, inferior and middle turbinates
E. pale pink mucous membrane, dry, matte, covered with yellow crusts

73. Such a paradoxical symptom as Paracusis Willisii (which consists in


improving hearing acuity when being in a noisy environment) is pathognamonic
for which disease?
A. for Meniere's disease
B. catarrh of the middle ear
C. otosclerosis
D. cochleoneuritis
E. eustacheitis
74. A 35-year-old patient complains of bilateral hearing loss, tinnitus. The disease
developed gradually over 3 years, notes an improvement in hearing in a noisy
environment. Otoscopy is normal. Rinne and Zhelle's tests are negative. What
disease are we talking about?
A. bilateral cochleoneuritis
B. otosclerosis
C. Meniere's disease
D. middle ear catarrh
E. eustachitis
75. A child developed a painful swelling above the external ear canal after acute
inflammation of the middle ear. Which group of mastoid cells is involved in
inflammation?
A. zygomatic cells
B. perianthral
C. apical
D. perisinous
E. perilabyrinth
76. A patient with middle ear inflammation developed facial asymmetry
in the form of lowering the angle of the mouth, smoothing of the nasolabial fold,
not closing the palpebral fissure on the affected side. What complication should
you think about?
A. thrombophlebitis of the sigmoid sinus
B. about otogenic arachnoiditis
C. about meningitis
D. about neuritis of the facial nerve
E. about the labyrinth

77. A patient in the absence of changes in the ear canal and on the eardrum has
complaints of ear congestion, hearing loss. What do you need to think about?
A. about purulent acute otitis media
B. about chronic suppurative disease of the middle ear
C. about inflammation of the labyrinth
D. about closing the auditory tube
E. about neuritis of the facial nerve

78. Patient K., 35 years old, during a strumectomy performed under local
anesthesia, had a sharp difficulty in breathing. Objectively: the patient is restless,
trying to get up; pale skin, acrocyanosis. Breathing is noisy, especially on
inhalation, 30 breaths per minute. A sharp retraction of the supra- and subclavian
fossa and intercostal spaces on inspiration. The voice is husky. Which of the
following diagnoses is the most likely?
A. cancer of the larynx. Stage III stenosis
B. allergic laryngeal edema. Stage III stenosis
C. traumatic paresis of the larynx.Stage III stenosis
D. acute infiltrative-edematous laryngitis. Stage III stenosis
E. diphtheria of the larynx. Stage III stenosis

79. In the evening, a 4-year-old child suddenly developed a convulsive cough,


shortness of breath and acrocyanosis while playing with buttons. After 2 minutes,
these phenomena disappeared. However, the mother brought the child to the ENT
department. Attacks of convulsive coughing recurred periodically. Which of the
following diagnoses is the most likely?
A. foreign body of the trachea
B. false croup
C. foreign body of the nasal cavity
D. acute laryngitis
E. allergic laryngitis

80. A 35-year-old man complained of progressive pain, redness, swelling of the


right auricle, body temperature 37.9. Two days ago there was an injury during
wrestling lessons. Objectively: the skin of the auricle is hyperemic, edematous,
except for the lobe, painful when touched; the external auditory canal is narrowed
at the entrance. Hearing within normal limits. Which of the following diagnoses is
the most likely?
A. eczema of the auricle
B. erysipelas of the auricle
C. hematoma of the auricle
D. perihondritis of the auricle
E. external diffuse otitis media
81. A 35-year-old Patient went to the emergency room complaining of pain and
swelling in the area of the right auricle. According to the patient, five days ago she
accidentally scratched her ear with a pin. After three days, there was a reddening of
the skin of the auricle, it increased in size, swollen, and a headache appeared. The
patient's condition gradually worsened, and the temperature has increased. Upon
examination, the skin of the auricle is sharply hyperemic, infiltrated. The process
extends to the parotid region, the skin of the auditory canal. Hyperemic areas are
surrounded by a line of demarcation. Make a diagnosis.
A. allergic dermatitis
B. mycotic lesion
C. esrysipelas
D. streptoderma
E. eczema

82. A patient of 30 years old, complained of hearing loss, noise in the right ear
after a runny nose. During otoscopy, the eardrum is retracted, gray in color, the
handle of the hammer is shortened along its course, the injection of blood vessels
is determined. The nasal mucosa is hyperemic, edematous, and the lower nasal
conchs are hypertrophied. Whisper speech in the right ear is 2 meters. Rinne's
experience is negative, lateralization of sounds in the affected ear. Make a
diagnosis.
A. acute otitis media on the right
B. acute tubotympanic right
C. adhesive otitis media on the right
D. acute sensorineural hearing loss on the right
E. outer limited otitis media on the right

83. In a 35-year-old patient with acute otitis media on the right, there was dizziness
with a sense of movement of objects counterclockwise, i.e. to the left, nausea and a
single vomiting. Spontaneous nystagmus to the right was detected. In the Romberg
position, the patient bends to the left. When performing finger - nose and finger -
finger probing, a left miss was detected. Upon examination of the ear, a purulent
discharge was detected in the auditory canal, a defect of the tympanic membrane in
the mesotimpanum, and a pulsating reflex was determined. During the hearing test,
a complete shutdown of the hearing function on the right was detected. What is the
complication of acute purulent otitis media?
A. Meniere's disease
B. otosclerosis
C. meningitis
D. labyrinthitis
E. mastoiditis
84. The patient 26 years old, taken to a hospital with complaints of dizziness,
single vomiting. The complaints appeared after the patient staggered to the left
during the ear cleaning in the clinic, spontaneous nystagmus and copious
perspiration appeared. The patient suffers from the pus out of the ear from
childhood, during conducting a Pressor test, nystagmus and dizziness appeared.
Otoscopy on the left: total defect of the tympanic membrane, pus in the tympanic
cavity, the medial wall is infiltrated, edematous, with areas of hyperplasia. What is
the complication of chronic purulent otitis media have this patient?
A. meningitis
B. brain abscess
C. external otitis media
D. labyrinthitis
E. mastoiditis

85. A 30-year-old Patient went to the clinic complaining of hearing loss and
tinnitus. According to the patient, hearing decreased gradually, but after childbirth,
the hearing loss became more noticeable. She marks a strange phenomenon when
she hears better in a noisy environment. During the examination of ENT organs, no
visible pathology was found. During otoscopy, the auditory passages are wide, no
sulfur masses are detected, and the tympanic membranes on both sides are pale, as
if thinned. Auditory function is impaired on both sides, but subjectively the patient
feels a greater decrease in hearing on the right. Make a diagnosis.
A. adhesive otitis media
B. tympanosclerosis
C. otosclerosis
D. Meniere's Disease
E. Corleonesi

86. The patient is 25 years, appealed with complaints of purulent discharge from
the ear, occasional headaches. Suffers from otitis since childhood, notes periodic
exacerbations, was not treated regularly. He hasn`t visited a doctor. During
otoscopy of the right ear, in the ear canal a small amount of pus was detected.
After the ear cleaning, a marginal defect of the tympanic membrane was found in
the posterior upper quadrant, filled with granulations. The tympanic membrane is
thickened, muddy, gray in color, with a small edge defect, and a thickened,
infiltrated edge. On the part of other ENT organs, no pronounced pathology was
detected. Make a diagnosis.
A. chronic mezotimpanit in the acute stage
B. chronic epitympanic complicated by granulation
C. fungal damage to the middle ear
D. bilateral acute purulent otitis media
E. chronic epimezotimpanit complicated cholesteatomas
87. Child, 10 years old, appealed with complaints about suppuration from the ear
pain and swelling in the ear area on the right. In the anamnesis: the child has
suffered for about three weeks. Despite the treatment, the pus remained, but the
pain was stopped. Upon examination, a plentiful purulent discharge in the right ear
canal is detected, a perforation in the mesotimpanum is detected, and an overhang
of posterosuperior wall of the ear canal in the bone part is determined. During
percussion of the posterior region on the right, child marks soreness. Make a
diagnosis.
A. acute purulent otitis media complicated by meningitis
B. chronic suppurative epimezotimpanit complicated by neuritis
C. acute purulent otitis media complicated by mastoiditis
D. exacerbation of chronic mesotympanitis
E. acute otitis externa complicated by acute cochleogram

88. A 30-year-old Patient went to the emergency room complaining of sharp,


unbearable pain in her right ear. According to the patient, she is ill for 4-5 days, the
disease began after hypothermia. The general condition is not affected, active, and
the temperature has not increased. Upon examination, the auricle is not changed,
but it is sharply painful when trying to pull the auricle back. The auditory canal is
narrowed, and there is a Muco-purulent discharge. It is difficult to pass only a
narrow funnel into the ear canal. The tympanic membrane is grey and muddy.
Make a diagnosis
A. furuncle of the ear canal on the right
B. erysipelas of the ear canal skin on the right
C. right-sided acute diffuse otitis media
D. acute otitis media on the right
E. acute parotid lymphadenitis on the right

89. A 42-year-old Patient complained of a sense of blocked right ear, noise in the
ear, and depression of hearing. For the first time he noticed it about a year ago, but
after self-blowing the auditory tube, the stuffiness of the ear was stopped.
Recently, this technique has ceased to help, hearing has not been restored, the
noise has become constant, and the feeling of a water overflow in the ear has
appeared. During otoscopy of the right ear, the tympanic membrane is gray,
muddy, and identification points are not visible. Rhinoscopy revealed a curvature
of the nasal septum, in the bone-cartilage part to the right side. Make a diagnosis.
A. adhesive otitis media
B. tympanosclerosis
C. serous tubotympanic
D. otosclerosis
E. Meniere's Disease
90. A patient 40 y.o, went to the ENT doctor with complaints of depression of
hearing, ringing in the ears. In the anamnesis: he had pneumonia and was treated
with antibacterial therapy. During the treatment period, there was periodic ringing
in the ears, after which he noted a decrease in hearing. Otoscopy: the eardrums are
grey, the identification points are preserved. Duringstudying hearing, whispered
speech in both ears is 3 meters, but there`s a decrease of speech intelligibility.
Acometry: Rinne's test is positive, the sample Shvabaha cropped on both sides.
there is no laterization in the Weber sample. Make a diagnosis.
A. tubotympanic
B. mucosal otitis media
C. corleonesi
D. otosclerosis
E. exudative otitis media

91. A patient of 20 years old, went to the clinic with complaints of tinnitus,
decrease of hearing on the right. When viewed from the right, the ear canal is free,
the tympanic membrane is dark black, in the lower parts: a small, lumpy formation.
During attempting a biopsy, profuse bleeding began, which was managed to be
stopped with the help of a tight tamponade. During palpating the parotid region, a
small painless formation is determined under the earlobe, a mildly elastic
consistency, not mobile, painless, the skin above it is not changed. Make a
diagnosis.
A. hemangioma of the middle ear
B. chemodectoma, glomus tumor
C. bleeding polyp of the middle ear
D. squamous cell carcinoma of the middle ear
E. acoustic neuroma

92. Patient 28 years old, appealed with complaints about permanent blocked of the
nose, which is relieved only after the instillation of naphazoline. Drops are used
throughout the year. The examination revealed hyperemia of the nasal mucosa,
hypertrophy of the conch shells. There is no purulent discharge in the nasal cavity.
Nasal septum along the midline. On retinogram sinuses there is the shading of all
sinuses but the intensity of the shading is not detected. Make a diagnosis:
A. allergic seasonal rhinitis
B. hypertrophy of the nasal conchs
C. medicamental rhinitis
D. vasomotor rhinitis, neurovegetative form
E. chronic subatrophic rhinitis

93. A patient of 23 years old, went to the clinic with complaints of difficult nasal
breathing. In the anamnesis: he had an injury about a week ago. There was no
nosebleed. He didn't lose consciousness. The nasal congestion increased gradually,
over several hours. At the same time, the patient's well-being did not change. Upon
examination, the nasal back is in the middle line, the skin of the face is covered in
places with hematomas, in the resolution stage, there is no nasal breathing. During
anterior rhinoscopy, the nasal septum is symmetrically thickened in the area of the
nasal vestibule, the conch shell and the nasal cavity aren`t seen. Make a diagnosis.
A. injury of the nose and soft tissues of the face
B. fracture of the nasal septum cartilage
C. nasal septum hematoma
D. dislocation of the quadrangular cartilage
E. chondroperichondritis of the nasal septum

94. A patient complains of constant the blocked of nasal, watery discharge,


sneezing, weakness. Ill for two weeks. He has noted the same condition the last
two or three years in a row at the same time of year. Upon examination, the nasal
mucosa is pale, copious watery discharge, and the nasal shells are swollen. There is
no purulent discharge. Make a diagnosis:
A. vasomotor rhinitis
B. acute rhinitis
C. allergic rhinitis
D. neurovegetative rhinitis
E. acute rhinosinusitis

95. A patient of 20 years old entered the clinic with complaints of difficult nasal
breathing, headache, weakness, subfebril temperature, purulent discharge from the
nose. She became very ill. In the clinic, a diagnosis was made: Acute purulent
maxillary sinusitis. A maxillary sinus puncture was performed, but no washing
water was obtained when the sinus contents were washed with a puncture needle.
The patient felt a sharp pain, immediately there was a swelling of the soft tissues of
the face. Name the complication of maxillary sinus puncture.
A. Air embolism.
B. Cheek puncture.
C. An eye socket puncture.
D. Hematoma of soft tissues of face
E. Cheek abscess

96. A 26-year-old Patient complained about the absence of nasal breathing in the
right side of the nose. This condition has been observed for the past two years.
Blocked of the nasal increased gradually, at first it was difficult to inhale from the
right side, then to exhale. Upon examination of the nasal cavity, moderate swelling
of the mucous membrane was detected, the nasal septum along the middle line. On
the radiograph, there is a darkening of the right maxillary sinus and cells of the
ethmoid bone. Upon examination of the nasopharynx, a pale gray formation with a
rounded shape obturates the choanae on the right Make a diagnosis.
A. exacerbation of chronic sinusitis on the right
B. polypous sinusitis
C. choanal polyp on the right
D. curvature of the nasal septum
E. angiofibroma of the nasopharynx

97. A 5-year-old Child went to the clinic complaining of difficulty on opening the
left eye, headache, blocked of the nasal, and malaise for four days. In the
anamnesis: he was ill acutely, after hypothermia there were pains in the area of the
frontal sinus and swelling of the left upper eyelid on the left. Objective: edema and
hyperemia of the upper eyelid on the left, the eye slit is narrowed, soreness on
palpation, in the left half of the nasal cavity there is a plentiful purulent discharge.
On the R-gram homogeneous darkening of the left maxillary sinus, in the frontal
sinus fluid level are detected Establish the nature of rhinogenic orbital
complication.
A. phlegmon of the orbit
B. retrobulbar abscess of the orbit
C. thrombosis of the eyelid orbit
D. periostitis of the eye socket
E. conjunctivitis

98. A 25-year-old woman applied for a consultation with complaints of headaches,


pain in the back of the head, sometimes of a girdling nature. There are no
complaints of difficult nasal breathing. Notes purulent discharge from the
nasopharynx, especially in the morning. Objectively: the nasal mucosa is
moderately hyperemic, the shells are not enlarged, and nasal breathing is free.
When examining the pharynx, there is a plentiful purulent discharge along the back
wall. On the review radiograph, the transparency of the paranasal sinuses is
preserved. Make a diagnosis.
A. ethmoiditis
B. frontal sinusitis
C. salpingitis
D. sphenoiditis
E. maxillary sinusitis

99. A 42-year-old Patient complained of drooping of the upper eyelid on the left,
difficult movingof the eyeball and diplopia when looking to the left. In the
anamnesis, periodic headaches, more in the occipital region, a feeling of blocked of
the nasal, a constant trickle of discharge along the back wall of the pharynx,
accordingly, he has to constantly expectorate phlegm, especially in the morning.
On CT in the axial and coronary projections, homogeneous darkening of the left
sphenoid sinus is determined. In addition, partial destruction of the intersinusal
septum of the main sinus was detected. Make a preliminary diagnosis
A. gemisinusitis on the left
B. purulent sphenoiditis
C. purulent ethmoiditis
D. cancer of the main sinus
E. purulent pansinusitis

100. Patient 35 years old has addressed to clinic with complaints of difficulty in
nasal breathing. In the anamnesis, at the place of residence she has a surgery-nasal
polypotomy. Nasal breathing was restored, but after three months the polyps
recurred. On the radiograph and CT scans of the paranasal sinuses darkening of
almost all the paranasal sinuses was revealed. The nasal cavity is obturated by
polyps. Nasal breathing is practically absent. Make a diagnosis.
A. polypous ethmoiditis
B. hyperplastic pansinusitis
C. the aspirin triad
D. polypose rhinosinusitis
E. allergic rhinitis

101. The patient is 14 years old addressed with complaints of constant the blocked
of the nasal, difficult nasal breathing. Periodically, the patient experiences
spontaneous nosebleeds, the cause of which the patient can not specify. The
examination revealed a moderately edematous nasal mucosa,
with a cyanotic tinge, without pathological discharge. Upon examination of the
nasopharynx, a lumpy formation covering the choanae was found. The formation
has a dark crimson color. Make a diagnosis.
A. angiofibroma of the nasopharynx
B. chemodectoma
C. adenoid vegetations
D. hemangioma
E. choanal polyp

102. Patient A., 25 years old, went to the clinic with complaints of hoarseness of
voice, a feeling of scratching, stinging and dryness in the throat. The onset of the
disease is associated with taking cold beer six days ago. According to the patient,
the disease began with the appearance of a dry cough, then a cough with sputum.
His well-being was fine, constantly felt discomfort when swallowing. During
laryngoscopy, hyperemia of the laryngeal mucosa was detected, more expressed in
the vocal folds, with areas of pinpoint hemorrhages. In the lumen of the larynx, a
small amount of viscous sputum, sometimes passing into crusts. Make a diagnosis.
A. acute catarrhal laryngitis
B. syphilis of the larynx (erythema stage)
C. tuberculosis of the larynx (the initial form)
D. laryngeal angina
E. edematous-infiltrative laryngitis

103. A patient of 30 years old, brought to the clinic with complaints of sharp pain
when swallowing, choking with liquid food, temperature increase to 38 ° C.
According to him, he was ill acutely, after ARI, and he didn`t observe bed rest. In
indirect laryngoscopy, the laryngeal mucosa is sharply hyperemic, edematous, and
infiltrated in the area of the scooplaryngeal folds and epiglottis. Due to infiltration,
the epiglottis is limited in mobility. The vocal folds are thickened, infiltrated, and
limited in mobility during phonation. Swelling of regional lymph nodes was
detected. Make a diagnosis.
A. phlegmonous laryngitis
B. abscess of the epiglottis
C. exacerbation of chronic pachydermic laryngitis
D. acute catarrhal laryngitis
E. edematous-infiltrative laryngitis

104. A patient 34 years old, taken to the emergency room by ambulance with
complaints of pain when swallowing, pains, difficult food intake, choking and even
not being able to swallow saliva.
In anamnesis, a week ago, he suffered a pharyngeal injury with a meat bone. After
that, there was pain when swallowing, which gradually increased, hoarseness of
the voice joined, shortness of breath appeared, and the temperature increased. In
indirect laryngoscopy, the epiglottis is sharply infiltrated, thickened, and almost
stationary. On the lingual surface of the epiglottis, there is an area of expressed
protrusion, in the center of which pus is visible. Make a diagnosis.
A. phlegmonous laryngitis
B. laryngeal angina
C. abscess of the epiglottis
D. chondrophora of the larynx
E. edematous-infiltrative laryngitis

105. A Child of 5 years of age has become acutely ill after suffering from acute
respiratory viral infections. Suddenly, in the middle of the night, woke up with a
paroxysmal barking cough. The child is restless, tossing in bed. Breathing is
whistling, sharply labored, inspiratory shortness of breath is expressed. The
retraction of the soft tissues of the jugular fossa, supra-and subclavian spaces, and
epigastric region is determined. This condition lasted for an hour, after which there
was profuse sweating, breathing improved and the child fell asleep. In the morning,
the child woke up almost healthy, but the voice was still somewhat hoarse. Make a
diagnosis.
A. diphtheria stenosis of the larynx, true croup
B. laryngeal stenosis of allergic origin
C. subglottic laryngitis, false croup
D. chronic stenosis of the larynx
E. edematous-infiltrative laryngitis

106. A 45-year-Old patient, a long-term smoker, suffers from hoarseness of voice.


Notes frequent exacerbations of laryngitis, which develops after local hypothermia
(cold drinking) and general. Upon examination of the larynx, areas of hypertrophy
of the mucous membrane in the interarytenoid space in the form of a transverse
roller with jagged edges protruding into the lumen of the larynx were revealed. The
vocal folds thickened, mobility is retained, but restricted, which is manifested in
not complete closing when he pronouns high-pitched sounds. Make a diagnosis
A. chronic atrophic laryngitis
B. chronic hyperplastic laryngitis
C. myasthenic paresis of the vocal folds
D. gipertoniya of fonotone
E. functional dysphonia

107. A 43-year-old patient went to the clinic complaining of hoarseness of voice,


shortness of breath during physical exertion, and a feeling of discomfort in the
larynx. In the anamnesis: having a strumectomy. In indirect laryngoscopy, the
glottis is free. The mucosa of the larynx is not changed, there is no pathological
discharge in the lumen of the larynx. During phonation, the left vocal fold is not
mobile, while the right one is still mobile. Make a diagnosis.
A. paralysis of the transverse laryngeal muscle
B. paralysis of the left recurrent nerve
C. central paralysis of the larynx
D. myopathic paralysis of the larynx
E. chronic myopathy

108. A 25-year-old patient was taken by ambulance to the emergency room,


complaining of shortness of breath and weakness. In the anamnesis, she suffered,
some intervention on the larynx is endolaryngeal, which she does not know.
Currently, the voice is rough, hoarse. There are manifestations of respiratory
failure of the first degree. A preliminary diagnosis of exacerbation of chronic
laryngitis. During endoscopy: pale gray formations in the form of a mulberry berry
on a wide base in the area of the anterior commissure and anterior third of the
vocal folds. Make a diagnosis.
A. larynx cancer
B. fibroma of the larynx
C. papillomas of the larynx
D. cysts of the larynx
E. chondrophora of the larynx
109. A 42-year-old patient turned to an ENT doctor with complaints of deafness
on the left, noise in the ear, and facial distortion. From history: received on the
treatment of neuritis of the auditory nerve on the left", but without effect. When
examining the left ear, the tympanic membrane is gray in color, with a pearlescent
tinge, the light cone is preserved. On the audiogram: increase of bone conduction
thresholds to 80 -90 DC, descending type to the left ear. There is no vestibular
excitability in the left ear, light trigeminal pain on the left and peripheral paresis of
the facial nerve on the left. Make a diagnosis.
A. neurinoma of the vestibulocochlear nerve
B. sensorineural left-sided deafness
C. chemodectoma or pomocna tumor of the middle ear
D. chronic labyrinthitis
E. cancer of the middle ear

110. An otorhinolaryngologist was contacted by a 62-year-old patient complaining


of formation in the ear region on the right. According to him, first noticed the
education about a year ago. First, a spot appeared, then a pimple, which over time
increased in size. After 7 to 8 months, the formation became ulcerated. Education
around the inflammatory infiltrate of the skin, in the form of oboda. When pressing
on the formation, there is a feeling of pain.
Make a diagnosis.
A. squamous cell carcinoma
B. basal cell carcinoma
C. cutaneous horn
D. atheroma
E. melanoma

111. A 45-year-old patient went to a therapist complaining of sore throat when


swallowing. Anti-inflammatory therapy was performed, but the condition
gradually worsened. The patient has a high fever, complains of General weakness,
headache, chills, sweating and insomnia. Upon examination, there is a spilled
hyperemia in the throat, tonsils are hypertrophic, purulent deposits are not found.
The skin and visible mucous membranes are pale. The liver, spleen, and axillary
lymph nodes are moderately enlarged. In the blood test - leukocytosis 112 x 9109 /
l, myeloblasts, lymphoblasts, hemocytoblasts predominate, erythrocytes 1.5 x 10
1/l, hemoglobin 33 g/l, color index 1, ESR 20 mm/h. Make a diagnosis.
A. mononuclear angina
B. acute myeloidleukemia
C. angina in tularemia
D. angina alimentary toxic aleukemia
E. agranulocytic angina
112. A 35-year-old patient went to the emergency room complaining of pain and
swelling in the area of the right auricle. According to the patient, five days ago she
accidentally scratched her ear with a pin. After three days, there was a reddening of
the skin of the auricle, it increased in size, swollen, and a headache appeared. The
patient's condition gradually worsened, and the temperature has increased. Upon
examination, the skin of the auricle is sharply hyperemic, infiltrated. The process
extends to the parotid region, the skin of the auditory canal. Hyperemic areas are
surrounded by a line of demarcation. Make a diagnosis.
A. allergic dermatitis
B. mycotic lesion
C. erysipelas
D. streptoderma
E. eczema

113. A patient of 30 years old, complained of hearing loss, noise in the right ear
after a runny nose. During otoscopy, the eardrum is retracted, gray in color, the
handle of the hammer is shortened along its course, the injection of blood vessels
is determined. The nasal mucosa is hyperemic, edematous, and the lower nasal
conchs are hypertrophied. Whisper speech in the right ear is 2 meters. Rinne's
experience is negative, lateralization of sounds in the affected ear. Make a
diagnosis.
A. acute otitis media on the right
B. acute tubotympanic right
C. adhesive otitis media on the right
D. acute sensorineural hearing loss on the right
E. outer limited otitis media on the right

114. In a 35-year-old patient with acute otitis media on the right, there was
dizziness with a sense of movement of objects counterclockwise, i.e. to the left,
nausea and a single vomiting. Spontaneous nystagmus to the right was detected. In
the Romberg position, the patient bends to the left. When performing finger - nose
and finger - finger probing, a left miss was detected. Upon examination of the ear,
a purulent discharge was detected in the auditory canal, a defect of the tympanic
membrane in the mesotimpanum, and a pulsating reflex was determined. During
the hearing test, a complete shutdown of the hearing function on the right was
detected. What is the complication of acute purulent otitis media?
A. Meniere's disease
B. otosclerosis
C. meningitis
D. labyrinthitis
E. mastoiditis
115. The patient 26 years old, taken to a hospital with complaints of dizziness,
single vomiting. The complaints appeared after the patient staggered to the left
during the ear cleaning in the clinic, spontaneous nystagmus and copious
perspiration appeared. The patient suffers from the pus out of the ear from
childhood, during conducting a Pressor test, nystagmus and dizziness appeared.
Otoscopy on the left: total defect of the tympanic membrane, pus in the tympanic
cavity, the medial wall is infiltrated, edematous, with areas of hyperplasia. What is
the complication of chronic purulent otitis media?
A. meningitis
B. brain abscess
C. external otitis media
D. labyrinthitis
E. mastoiditis

116. A 30-year-old patient went to the clinic complaining of hearing loss and
tinnitus. According to the patient, hearing decreased gradually, but after childbirth,
the hearing loss became more noticeable. She marks a strange phenomenon when
she hears better in a noisy environment. During the examination of ENT organs, no
visible pathology was found. During otoscopy, the auditory passages are wide, no
sulfur masses are detected, and the tympanic membranes on both sides are pale, as
if thinned. Auditory function is impaired on both sides, but subjectively the patient
feels a greater decrease in hearing on the right. Make a diagnosis.
A. adhesive otitis media
B. tympanosclerosis
C. otosclerosis
D. Meniere's Disease
E. Corleonesi

117. The patient is 25 years, appealed with complaints of purulent discharge from
the ear, occasional headaches. Suffers from otitis since childhood, notes periodic
exacerbations, was not treated regularly. He hasn`t visited a doctor. During
otoscopy of the right ear, in the ear canal a small amount of pus was detected.
After the ear cleaning, a marginal defect of the tympanic membrane was found in
the posterior upper quadrant, filled with granulations. The tympanic membrane is
thickened, muddy, gray in color, with a small edge defect, and a thickened,
infiltrated edge. On the part of other ENT organs, no pronounced pathology was
detected. Make a diagnosis.
A. chronic mezotimpanit in the acute stage
B. chronic epitympanit, complicated by granulation
C. fungal damage to the middle ear
D. bilateral acute purulent otitis media
E. chronic epimezotimpanit complicated cholesteatomas
118. Child, 10 years old, appealed with complaints about suppuration from the ear
pain and swelling in the ear area on the right. In the anamnesis: the child has
suffered for about three weeks. Despite the treatment, the pus remained, but the
pain was stopped. Upon examination, a plentiful purulent discharge in the right ear
canal is detected, a perforation in the mesotimpanum is detected, and an overhang
of posterosuperior wall of the ear canal in the bone part is determined. During
percussion of the posterior region on the right, child marks soreness. Make a
diagnosis.
A. acute purulent otitis media complicated by meningitis
B. chronic suppurative epimezotimpanit complicated by neuritis
C. acute purulent otitis media complicated by mastoiditis
D. eacerbation of chronic mesotympanitis
E. acute otitis externa complicated by acute cochleogram

119. A 30-year-old patient went to the emergency room complaining of sharp,


unbearable pain in her right ear. According to the patient, she is ill for 4-5 days, the
disease began after hypothermia. The general condition is not affected, active, and
the temperature has not increased. Upon examination, the auricle is not changed,
but it is sharply painful when trying to pull the auricle back. The auditory canal is
narrowed, and there is a Muco-purulent discharge. It is difficult to pass only a
narrow funnel into the ear canal. The tympanic membrane is grey and muddy.
Make a diagnosis
A. furuncle of the ear canal on the right
B. erysipelas of the ear canal skin on the right
C. right-sided acute diffuse otitis media
D. acute otitis media on the right
E. acute parotid lymphadenitis on the right

120. A 42-year-old patient complained of a sense of blocked right ear, noise in the
ear, and depression of hearing. For the first time he noticed it about a year ago, but
after self-blowing the auditory tube, the stuffiness of the ear was stopped.
Recently, this technique has ceased to help, hearing has not been restored, the
noise has become constant, and the feeling of a water overflow in the ear has
appeared. During otoscopy of the right ear, the tympanic membrane is gray,
muddy, and identification points are not visible. Rhinoscopy revealed a curvature
of the nasal septum, in the bone-cartilage part to the right side. Make a diagnosis.
A. adhesive otitis media
B. tympanosclerosis
C. serous tubotympanic
D. otosclerosis
E. Meniere's Disease
121. Patient 0f 40 years old, went to the ENT doctor with complaints of
depression of hearing, ringing in the ears. In the anamnesis: he had pneumonia and
was treated with antibacterial therapy. During the treatment period, there was
periodic ringing in the ears, after which he noted a decrease in hearing. Otoscopy:
the eardrums are grey of colour, the identification points are preserved.
Duringstudying hearing, whispered speech in both ears is 3 meters, but there`s a
decrease of speech intelligibility. Acumetry: Rinne's test is positive, the sample
Shvabaha cropped on both sides. there is no laterization in the Weber sample.
Make a diagnosis.
A. tubotympanic
B. mucosal otitis media
C. corleonesi
D. otosclerosis
E. exudative otitis media

122. A Patient of 20 years old, went to the clinic with complaints of tinnitus,
decrease of hearing on the right. When viewed from the right, the ear canal is free,
the tympanic membrane is dark black, in the lower parts: a small, lumpy formation.
During attempting a biopsy, profuse bleeding began, which was managed to be
stopped with the help of a tight tamponade. During palpating the parotid region, a
small painless formation is determined under the earlobe, a mildly elastic
consistency, not mobile, painless, the skin above it is not changed. Make a
diagnosis.
A. hemangioma of the middle ear
B. chemodectoma, glomus tumor
C. bleeding polyp of the middle ear
D. squamous cell carcinoma of the middle ear
E. acoustic neuroma

123. Patient 28 years old, appealed with complaints about permanent blocked of
the nose, which is relieved only after the instillation of naphazoline. Drops are
used throughout the year. The examination revealed hyperemia of the nasal
mucosa, hypertrophy of the conch shells. There is no purulent discharge in the
nasal cavity. Nasal septum along the midline. On retinogram sinuses there is the
shading of all sinuses but the intensity of the shading is not detected. Make a
diagnosis:
A. allergic seasonal rhinitis
B. hypertrophy of the nasal conchs
C. medicamental rhinitis
D. vasomotor rhinitis, neurovegetative form
E. chronic subatrophic rhinitis.
124. A patient of 23 years old, went to the clinic with complaints of difficult nasal
breathing. In the anamnesis: he had an injury about a week ago. There was no
nosebleed. He didn't lose consciousness. The nasal congestion increased gradually,
over several hours. At the same time, the patient's well-being did not change. Upon
examination, the nasal back is in the middle line, the skin of the face is covered in
places with hematomas, in the resolution stage, there is no nasal breathing. During
anterior rhinoscopy, the nasal septum is symmetrically thickened in the area of the
nasal vestibule, the conch shell and the nasal cavity aren`t see. Make a diagnosis.
A. Injury of the nose and soft tissues of the face
B. Fracture of the nasal septum cartilage
C. Nasal septum hematoma
D. Dislocation of the quadrangular cartilage
E. Сhondroperichondritis of the nasal septum

125. The Patient complains of constant the blocked of nasal, watery discharge,
sneezing, weakness. Ill for two weeks. He has noted the same condition the last
two or three years in a row at the same time of year. Upon examination, the nasal
mucosa is pale, copious watery discharge, and the nasal shells are swollen. There is
no purulent discharge. Make a diagnosis:
A. vasomotor rhinitis
B. acute rhinitis
C. allergic rhinitis
D. neurovegetative rhinitis
E. acute rhinosinusitis.

126. A Patient of 20 years old entered the clinic with complaints of difficult nasal
breathing, headache, weakness, subfebril temperature, purulent discharge from the
nose. She became very ill. In the clinic, a diagnosis was made: Acute purulent
maxillary sinusitis. A maxillary sinus puncture was performed, but no washing
water was obtained when the sinus contents were washed with a puncture needle.
The patient felt a sharp pain, immediately there was a swelling of the soft tissues of
the face. Name the complication of maxillary sinus puncture.
A. air embolism
B. cheek puncture
C. an eye socket puncture
D. hematoma of soft tissues of face
E. cheek abscess

127. A 18-year-old female patient complained of minor bleeding from the nasal
cavity almost daily, more often in the morning hours. At the time of examination,
there was no bleeding. Hemodynamic parameters within the normal range. With
anterior rhinoscopy in the anterior sections of the nasal septum, an expansion of
the vascular pattern is noted, in the nasal passages there is a small amount of
hemorrhagic clots.
Which of the following diagnoses is the most likely?
A. bleeding polyp of the nasal cavity
B. juvenile angiofibroma of the nasopharynx
C. recurrent epistaxis
D. polysinusitis
E. neoplasm of the nasal cavity

128. Patient K. 23 years old, complains of a stuffy nose, hearing loss, tinnitus.
Anamnesis: she suffered an acute respiratory illness about a week ago. On
examination: AD/AS-eardrums slightly pink, injection of vessels along the handle
of the hammer. SR-4m, the nasal mucosa is hyperemic, edematous, Muco-purulent
discharge. What is the diagnosis:
A. tubotympanic
B. bullous otitis media
C. Adhesive otitis media
D. Idiopathic genitempo
E. Tantric

129. A 26-year-old patient went to a polyclinic complaining of severe itching in


the ears, which periodically turns into a feeling of pain. History: neodnakratno
aggravation in the form of diffuse external otitis. During otoscopy: AD-AS-the
auditory passages are filled with plates of dry epidermis, sometimes slushivayusya
in the form of casts. After their removal, the eardrums are gray in color on both
sides without identifying points, slightly as if covered with "fluff". What is the
diagnosis:
A. eczema of the ear passages
B. external diffuse otitis media
C. furuncle of the auditory passages
D. fungal external otitis media
E. cystosis of the auditory passages

130. Patient G. 45 years old, went to the doctor with complaints of a feeling of
congestion of the left ear, hearing loss, tinnitus (low-frequency character), a sense
of fluid transfusion in the ear, autophony. During otoscopy: AS - the auditory canal
is free, the gray eardrum is slightly bulging. SHR - 3M. Rhinoscopy: the nasal
mucosa is subatrophic, the nasal septum is curved to the left, in the form of a bony
ridge at the level of the lower nasal passage. What is the diagnosis:
A. adhesive otitis media
B. acute otitis media
C. eustachitis
D. mucosal otitis media
E. bullous viral otitis media

131. A 30-year-old patient went to a doctor complaining of severe pain and


congestion in her right ear. From anamnesis suffers within 3 days after an acute
respiratory disease. Pain in the ear appeared suddenly in the night, gave up in the
throat, in the head. During otoscopy: AD-the auditory canal is free, the eardrum is
sharply hyperemic, bulges out, and there are no identifying contours. Rhinoscopy:
the nasal mucosa is moderately edematous, Muco-purulent discharge in the
General nasal passage. What is the diagnosis:
A. acute mastoiditis
B. external otitis media
C. adhesive otitis media
D. acute otitis media
E. eustachitis

132. A 31-year-old patient went to the emergency room complaining of severe


pain in her right ear. He considers himself ill for 3 days, after an acute respiratory
viral infection. During otoscopy: AD-the eardrum is hyperemic, edematous,
covered with blood vesicles filled with hemorrhagic contents. What is the
diagnosis?
A. acute otitis media bullosa
B. acute otitis media
C. mucosal otitis media
D. external diffuse otitis media
E. otomycosis

133. A 28-year-old patient went to the emergency room complaining of severe pain
in the left ear, radiating to the temporal and parietal areas, increasing with
chewing, and an increase in temperature to 37.7 C. During otoscopy: AS -in the ear
canal - on the anterior wall, a conical elevation is determined, the skin on its
surface is hyperemic. In the center of the formation - a purulent head, the lumen of
the auditory canal is narrowed, the eardrum is not visualized. An enlarged lymph
node is palpated in front of the ear. What is the diagnosis:
A. furuncle of the ear canal
B. acute otitis media
C. mucosal otitis media
D. external diffuse otitis media
E. fungal otitis media
134. A patient is 36 years old, went to emergency room with complaints of pain in
the ears, discharge of purulent character. It does not mark a decrease in hearing.
The overall condition is satisfactory, the temperature is subfebrile. The disease
started about a week ago. The onset of the disease is associated with the traumatic
removal of sulfur plugs (hairpin). He was treated independently at home, but
without any effect. During otoscopy: AD-AS-the lumen of the auditory passages is
narrowed, filled with a lump-like purulent discharge, after the toilet it was only
possible to enter a narrow funnel. What is the diagnosis?
A. external diffuse otitis media
B. dry eczema of the external auditory passages
C. Exacerbation of chronic otitis media
D. mucosal otitis media
E. otomycosis

135. A patient of 20 years old, complains of severe pain in the right ear, radiating
to the temple, ear congestion, headache, an increase in body temperature to 38
degrees. From anamnesis-ill for 2 days. The onset of the disease is associated with
a runny nose. During otoscopy: the AD-auditory canal is free, the skin is not
changed, the eardrum is vividly hyperemic, infiltrated, protruding, there are no
identifying points. Hearing in the right ear is reduced by SR-2m, Rinne's
experience is negative, and sounds are lateralized to the right. What is the
diagnosis?
A. adhesive otitis media
B. meringa
C. stageit
D. exudative otitis media
E. acute otitis media

136. A 10-year-old patient complained of pain in the right ear, suppuration,


headache, and hearing loss. He was treated on an outpatient basis for two weeks,
but without any effect. Two days ago, there was pain in the behind-the-ear area, the
temperature is subfebrile, pus from the ear has increased. Objectively: AD-defined
swelling in the posterior region, smoothed out the posterior fold, palpation pain in
the projection of the mastoid process. In the ear canal, there is a plentiful Muco-
purulent discharge, in the bone Department, an overhang of the posterior-upper
wall. Make a diagnosis:
A. furuncle of the external auditory canal
B. acute middle purulent otitis
C. exacerbation of chronic purulent otitis media
D. acute purulent otitis media
E. acute -necrotic otitis media
137. A 35-year-old patient was taken to an ENT clinic with complaints of severe
systemic vertigo, nausea, vomiting, ringing in the left ear, and balance disorders. In
the anamnesis, similar attacks were observed in the patient during the last five
years, with each attack marks a gradual decrease in hearing. This attack began
suddenly and for no apparent reason. AD and AS without visible pathology. Make
a diagnosis:
A. purulent labyrinthitis
B. acute otitis media
C. Meniere's Disease
D. fistula of the labyrinth
E. otosclerosis

138. A 45-year-old patient turned to an ENT doctor of a polyclinic with


complaints of: periodic nasal congestion, which appeared after a nose injury. The
patient presented fairly characteristic complaints: if he lay on his right side, the
right side of the nose was blocked; if he lay on his left side, the left side of the nose
was blocked. During rhinoscopy: the nasal septum is curved to the right, the nasal
passage is narrowed to the right, the lower nasal conchae are moderately
hypertrophied, the nasal mucosa is pale pink with a cyanotic tinge.
Make a diagnosis:
A. drug-induced rhinitis
B. seasonal allergic rhinitis
C. allergic year-round rhinitis
D. vasomotor rhinitis
E. hypertrophic rhinitis

139. A 27-year-old patient went to an ENT doctor of a polyclinic with complaints


of: weakness, discomfort, nasal congestion and sneezing attacks. On examination:
the nasal mucosa is pale with a gray tinge, the discharge is serous, abundant and
watery. According to the patient, this condition is observed at the same time of
year during the last three years. Moreover, the process is becoming longer and
harder every year.
Make a diagnosis:
A. drug-induced rhinitis
B. vasomotor rhinitis
C. year-round allergic rhinitis
D. seasonal allergic rhinitis
E. neurovegetative runny nose

140. A 33-year-old patient went to an ENT doctor complaining of difficulty in


nasal breathing, weakness, and headaches. From the medical history of the patient
for several months did not apply to doctors. During anterior rhinoscopy, pale gray
mobile formations of a rounded shape with a smooth surface are identified, which
obturate the common nasal passage on both sides. Nasal breathing is difficult.
Make a diagnosis:
A. nasal polyps
B. the aspirin triad
C. hypertrophic rhinitis
D. choanal polyp
E. hypertrophy of nasal conchs

Application tests.

1. A 64-year-old female patient consulted an ENT doctor with complaints of


hearing loss in the right ear. During otoscopy, a dense yellow-brown mass was
found in the external auditory canal, completely obstructing it. The left external
auditory canal is free. A preliminary diagnosis was made: Sulfuric plug of the ear
canal on the right. Which of the following is the best way to remove the sulfur
plug?

A. removal by flushing
B. removal with tweezers
C. removal with a loop
D. removal with a hook
E. removal by clamp

2. A 23-year-old patient consulted an ENT doctor with complaints of congestion in


the left ear, hearing loss and pain in the left ear. He associates his disease with
hypothermia. The pain in the ear was constant and sharply intensified with
pressure on the tragus. When otoscopy: the left external auditory canal is evenly
narrowed due to edema and infiltration of the skin of its walls, the tympanic
membrane is immense. There is no exudate in the external auditory canal. Make a
preliminary diagnosis and prescribe treatment.

A. acute otitis media, paracentesis of the tympanic membrane


B. external diffuse otitis media, antibiotic therapy
C. exudative otitis media, catheterization of the auditory tube
D. adhesive otitis media, ear blowing according to Politzer
E. furuncle of the external passage, surgical treatment

3. A 25-year-old patient, after an attempt to remove sulfur from the right external
auditory canal, began to complain of constant pain in the ear. Within 2 days, the
pain gradually increased, becoming especially acute when the mouth was opened.
The pressure on the tragus was also painful. With otoscopy, an inflammatory
infiltrate of a round shape is determined in the area of the entrance to the external
auditory canal on the right. Your opinion on the nature of the pathological process
and its localization within the external auditory canal.

A. furuncle in the anterior wall


B. furuncle in the area of the lower wall
C. furuncle in the area of the upper wall
D. furuncle in the area of the inner wall
E. furuncle in the back wall

4. Child, 6 months old, became restless, often woke up at night, cried. The
pediatrician called to the house examined the child and revealed the following
signs of the disease. Body temperature - 37.0 °, nasal breathing is disturbed, crying
and anxiety of the child increases with gentle, light pressure on the tragus. No
other pathological changes were found. The doctor sent the child to the ENT
department, where he was given a presumptive diagnosis: Bilateral tubootitis.
Which of the following drugs in the form of nasal drops is more appropriate to use
in this situation?

A. naphthyzine
B. sanorin
C. galazolin
D. protargol
E. aqua Maris

5. A 57-year-old man complains of constant itching in the external auditory canals,


which sometimes becomes unbearable. Manipulating various objects in the ears
(matches, etc.) does not bring relief. Moreover, twice, after such manipulations,
there was acute diffuse otitis externa (diagnosed by a polyclinic doctor) with sharp
pains in the ears and high fever. A preliminary diagnosis was made: Otomycosis.
What diagnostic methods of research should be used to confirm the diagnosis?

A. cytology
B. otoscopy
C. acumetry
D. audiometry
E. tympanometry

6. A child of 8 years old, on the 3rd day of a sluggish ARVI developed severe
shooting pains in the right ear. After taking analgin, the pain stopped, but resumed
after 2 hours, nausea and single vomiting appeared. After examination of the ENT
doctor, a clinical diagnosis was made: Acute otitis media, stage of pre-perforative
purulent inflammation on the right. Which one of the following treatment options
is most appropriate in this situation?

A. antibiotic therapy
B. antihistamines
C. local therapy
D. physiotherapy
E. paracentesis

7. An 8-year-old girl, in the midst of ARVI, developed a feeling of stuffiness in her


left ear, which was soon replaced by a throbbing tearing pain that intensified when
swallowing. The compress and the infusion of drops into the ear slightly improved
the child's condition. However, after 3 hours, the pain in the ear intensified,
acquired a bursting character, and therefore, the child was delivered to the ENT
doctor by the SP machine. Diagnosed with Acute otitis media, stage of pre-
perforative purulent inflammation on the left. Which one of the following
treatment options is most appropriate in this situation?

A. antibiotic therapy
B. antihistamines
C. local therapy
D. physiotherapy
E. paracentesis

8. A 5-year-old child complains of shooting pains in the right ear. The parents
treated the child with home remedies - after the use of thermal procedures and
analgesics, the pain in the ear stopped. However, a day later the pain in the ear
resumed, acquired a bursting, then tearing character. The body temperature rose to
38 °, a headache joined, and there was a single vomiting. The child was delivered
to the ENT department by the SP car. The doctor examined the patient and made a
clinical diagnosis: Acute purulent otitis media on the right. What is the most
effective treatment for improving a child's condition?

A. antibiotic therapy
B. antihistamines
C. local therapy
D. physiotherapy
E. paracentesis

9. A 3-year-old child, against the background of acute rhinitis, has a temperature


rise of up to 38.2 °, anxiety, tearfulness appeared. While crying, he held onto his
left ear. The grandmother treated the child with aspirin and a heating pad on the
left ear. After home medical procedures, the child's condition improved, he calmed
down, the temperature dropped to 37.4 °. On the 3rd day, against the background
of minor pains in the ear, there appeared pasty (edema) of the skin behind the ear.
This alarmed the parents, and they turned to the ENT doctor of the hospital, who
hospitalized the child, saying that he would try to limit himself to minor surgical
intervention. What kind of surgical intervention were you talking about?

A. sanitizing radical surgery


B. antrotomy
C. atticoanthrotomy
D. atticoanthromastoidotomy
E. paracentesis

10. A 28-year-old man has a sudden rise in temperature up to 39 °, chills, aching


pain in joints and muscles, which is why the patient took Fervex. The patient's
condition improved, but the next day the temperature rose again to 37.5 °, there
was shooting, then bursting pain and noise in the left ear, hearing decreased. The
therapist summoned to the house diagnosed "flu" and referred the patient to the
ENT doctor, who, after examination, diagnosed: Acute otitis media complicated by
cochleoneuritis on the left. What diagnostic method was used in this case?

A. tympanometry
B. acoustic impedance measurement
C. tonal audiometry
D. vestibulometry
E. research in whispering

11. A 9-year-old child has had right-sided acute purulent otitis media 3 times in the
last year. Twice he was treated on an outpatient basis, the last time the otitis media
was difficult, I had to do paracentesis in the hospital. In the study of hearing
revealed bilateral hearing loss of the type of impaired sound conduction. From the
anamnesis, it was possible to find out that the child often and for a long time
suffers from colds, for a long time his nasal breathing is disturbed, he does not
study well. What additional research methods need to be used to clarify the
diagnosis?
A. hypopharyngoscopy
B. mesopharyngoscopy
C. epipharyngoscopy
D. anterior rhinoscopy
E. superior rhinoscopy

12. A 12-year-old child has shooting pains in the ear against the background of an
acute rhinitis. After taking analgin, the pain decreased, but after a few hours it
resumed and intensified, became constant, acquired a tearing character. The patient
was delivered to the ENT department by the SP car. On examination: temperature
37.8 °. The tragus symptom is negative. The external auditory canal is free. The
tympanic membrane is hyperemic, infiltrated, there are no identifying signs. There
is a protrusion in the anterior sections of the membrane, clinical diagnosis: Acute
otitis media. Define the stage of the disease and treatment tactics?

A. eustachitis, antihistamines, vasoconstrictor drops and physiotherapy


B. catarrhal, antibiotics, antihistamines, vasoconstrictor drops
C. preperforative, paracentesis, antibiotics, analgesics, antihistamines
D. postperforative antihistamines, vasoconstrictor drops and physiotherapy
E. reparative, local antiseptics, physiotherapy

13. A 25-year-old woman, after suffering an acute respiratory viral infection,


developed an ear pain, first shooting, then bursting. A day later, the pain subsided,
but there was suppuration from the ear, hearing loss and tinnitus. With these
complaints, the patient turned to the ENT doctor of the polyclinic. Which of the
tuning fork studies will help determine by what type of hearing impairment?

A. jelle's Experience
B. federici's experience
C. schwabach experience
D. bing's experience
E. weber's experiment

14. A 10-year-old child complained of swelling and shooting pains in the left ear
within a week. On examination: the protrusion of the left auricle is noteworthy, the
furrow behind the ear is smoothed. Palpation behind the ear is sharply painful,
especially in the apex of the mastoid process. During otoscopy, the external
auditory canal is filled with purulent exudate. Immediately after washing, the
external auditory canal is filled with exudate. There is also some narrowing of the
external auditory canal due to the infiltration of the skin of its posterior-superior
wall in the bone region. Make a diagnosis.

A. mastoiditis
B. abscessing furuncle of the ear
C. behind the ear lymphadenitis
D. parotitis
E. external diffuse otitis media

15. A 5-year-old child was taken to the ENT hospital with complaints of shooting
pains and swelling in the left ear. On examination: the left auricle is protruded. In
the area behind the ear, a fluctuating protrusion of soft tissues the size of a chicken
egg, edema and infiltration of the skin, extending to the parietal, temporal and
preauricular regions, are determined. Palpation behind the ear is sharply painful.
The external auditory canal has a slit shape due to infiltration of the skin of the
posterior-superior wall. The eardrum is poorly visible. Diagnosed with Mastoiditis.
Which of the following interventions is more appropriate to apply in this situation?

A. sanitizing radical surgery


B. antrotomy
C. atticoanthrotomy
D. atticoanthromastoidotomy
E. antromastoidotomy

16. Patient 47 years old, turned to the ENT doctor of the polyclinic with
complaints of constant suppuration from the right ear. Suppuration arose a month
ago after suffering a respiratory viral disease. From the anamnesis it turned out that
he had been suffering from suppuration from the ear since childhood. When
otoscopy on the right: in the external auditory canal, a scanty mucopurulent
exudate is determined. In the stretched part of the tympanic membrane at the
border of the anterior-inferior and anteroposterior quadrants, there is a rounded
perforation with a diameter of 3 mm. Which of the following survey methods
should be done first?
A. R- gr
B. CT
C. MRI
D. audiometry
E. take a swab
17. A 45-year-old patient has been suffering from chronic purulent otitis media
since childhood. Exacerbations are rare - approximately 1-2 times a year. The
patient turned to the ENT doctor of the polyclinic about another exacerbation of
the disease. During otoscopy: scanty purulent exudate in the external auditory
canal. The tympanic membrane is cicatricially changed; in the antero-superior
quadrant, a small perforation with a diameter of about 1 mm is determined. From
above, the perforation is limited by the upper wall of the external auditory canal.
When probing the perforation with the Voyachek probe, the tip of the probe
palpates the rough bone surface. Make a clinical diagnosis and prescribe treatment.

A. chronic mesotympanitis, conservative


B. chronic mesotympanitis, surgical
C. chronic epitympanitis, medication
D. chronic epitympanitis, surgical
E. chronic epimesotympanitis, conservative
18. Patient 35 years old, turned to the ENT doctor of the polyclinic about the
exacerbation of right-sided chronic suppurative otitis media. Considers himself
sick for 5 years. Exacerbations occur 3-4 times a year, provoked by a respiratory
viral infection. When otoscopy in the right external auditory canal - a small amount
of purulent exudate. The tympanic membrane is dull, the identification signs are
blurred. In the anterior-upper quadrant, perforation is determined, the lumen of
which is filled with gray-white structureless masses. Make a clinical diagnosis,
identify additional research methods that can be applied in this case.
A. chronic mesotympanitis, ear swab
B. chronic mesotympanitis, acumetry
C. chronic epitympanitis, tympanometry
D. chronic epitympanitis, CT of the temporal bone
E. chronic mesotympanitis R- gr. mastoid

19. When examining conscripts at the military registration and enlistment office, a
doctor suspected a simulation of deafness. Otoscopically no pathology was
revealed. Which of the following studies will objectively assess the condition of
the auditory analyzer?
A. computer audiometry
B. acumetry
C. chamber research
D. mastoid radiography
E. study of the patency of the Eustachian tube

20. After suffering a severe flu, a patient has a significant hearing loss in the right
ear. Acute neuritis of the auditory nerve is suspected.Select which audiometric data
will confirm this diagnosis?
A. reduction of sound conduction
B. decrease in sound conduction and sound perception
C. decrease in sound perception
D. no change
E. enhancement of sound perception
21. A 27-year-old patient complains of hearing loss, tinnitus, more pronounced on
the right. In a noisy environment he periodically notes some improvement in
hearing. He cannot indicate the causes of the disease. The development of the
disease is gradual. The hearing loss and tinnitus worsened significantly a year ago
during pregnancy. Objectively: the auricles and areas of the mastoid processes are
not externally changed, painless on palpation. The ear canals are wide. The
eardrums are gray, shiny, with pronounced identification marks. After
examination, the patient was diagnosed with otosclerosis. Choose one of the main
types of ear surgery for otosclerosis?
A. paracentesis
B. antrotomy
C. radical middle ear surgery
D. stapedoplasty
E. mastoidotomy
22. A patient working as a drifter in a mine complains of hearing loss and tinnitus.
The general condition is not disturbed, the body temperature is normal. These
complaints are disturbing throughout the year and progress. Objectively: in the
auditory canals on both sides there is a parietal accumulation of sulfur, the
tympanic membranes are thin, gray, with clear contours, and mobile. Hearing: SR
on both sides - 4 m. Rinne's experience is small positive. Schwabach's experience
is shortened on both sides. Professional cochleoneuritis is suspected. Which of the
following research methods is the most important for making a diagnosis?
A. tuning fork examination of hearing
B. х-ray of the mastoid processes
C. audiometry
E. determination of the patency of the auditory tubes
E. tympanometry
23. A 32-year-old patient came to the clinic with complaints of hearing impairment
in both ears, a feeling of ear congestion, autophony, a feeling of fluid transfusion in
the ears when the head position was changed. Ill for a month after suffering an
acute respiratory illness. Otoscopy: the tympanic membranes are gray, turbid,
identification points are not determined, the mobility of the tympanic membranes
is limited. The patency of the auditory tubes is III degree. ShR on the right ear - 2
m, on the left ear - 2.5 m, PP - 5 m on both ears. Was diagnosed with exudative
otitis media. Choose from the following method of blowing through the auditory
tubes to inject medication into the tympanic cavity:
A. valsalva method
B. politzer's method
C. catheterization
D. toynbee method
E. empty sip method

24. Prescription of which drug from the listed ones, which is mandatory in patients
with acute ear noise, against the background of acute sensorineural hearing loss
A. systemic glucocorticosteroids
B.topic glucocorticosteroids
C. local anesthetic behind the ear block
D. ACE inhibitor
E. vitamin therapy
25. Features of drug therapy for patients with chronic tinnitus are:
A. rapid onset of effect and short course of treatment (up to 10 days)
B. large doses of the drug and a quick onset of effect
C. long-term use of drugs and the onset of the effect not earlier than 20 days
D. single drug monotherapy and rapid onset of effect
E. short course of treatment with one drug

26. A patient with otosclerosis has a negative Zhelle test. What does Zelle's
experience with the C128 tuning fork imply:
A. Changing the volume of the tuning fork when the pressure in the ear canal
changes
B. no change in the volume of the tuning fork when the pressure in the ear canal
changes
C. the appearance of ringing when the pressure in the ear canal changes
D. the appearance of whistling and ringing when the pressure in the ear canal
changes
E. the appearance of dizziness when the pressure in the ear canal changes

27. Choose from the following conservative treatments for middle ear catarrh:
A. through a catheter, the introduction of corticosteroids, adrenomimetics
into the auditory tube
B. physiotherapy - UHF in the ear, KUF
C. compresses on the parotid region, alcohol drops in the ear
D. bypass grafting of the tympanic membrane
E. adenotomy

28. What medicinal substances cause sensorineural hearing loss?


A. aminoglycosides
B. cephalosporins
C. aminopenicillins
D. macrolides
E. preparations of calcium, potassium

29. Which of the following is the rehabilitation of hearing in patients with


sensorineural deafness?
A. perform cochlear implantation
B. using hearing aids
C. implanting ossicular prostheses
D. tympanoplasty
E. conservative treatment

30. Choose what drug is injected into the tympanic cavity according to the Lang
method for Meniere's disease?
A. 0.3-0.5 ml of gentamicin
B 1.0 ml penicillin
C. 0.5 ml of hydrocortisone
D. 1 ml streptomycin
E.1ml ceftriaxone
31. Cochlear implantation is indicated for:
A. conductive hearing loss 1 and 2 degrees
B. mixed hearing loss
C. sensorineural hearing loss of 1 and 2 degrees
D. complete loss of auditory function
E. conductive hearing loss grade 3

32. Specify a drug that improves the conduction of impulses in the cholinergic
synapses of the auditory system, used for sensorineural hearing loss?
A. trental
B. aloe extract
C. galantamine
D.tanakan
E. lidase
33. A 35-year-old patient complains of recurrent episodes of systemic vertigo
among complete health against the background of growing noise and hearing loss
in the left ear. Dizziness attacks are accompanied by nausea, vomiting, balance
disorder: the patient cannot move, every attempt to move and change the position
of the head in space increases dizziness and autonomic disorders. At the time of an
attack, blood pressure is unstable: max 140 / 90-100, min. - 90/50 mm Hg. the
attack lasts up to 4-6 hours, goes away on its own. Outside the attack, the state of
complete disability, on the audiogram, sensorineural hearing loss of 3-4 degrees on
both sides. About 10 years ago, the diagnosis was made - Meniere's disease.
Choose which surgical intervention is more expedient for this patient?
A. opening (decompression) of the endolymphatic sac
B. antrotomy
C. transtympanic injection of hydrocortisone
D. myringoplasty
E. stapedoplasty

34.Specify the name of the operation for exudative otitis media:


A. antromastoidotomy
B. radical surgery on the temporal bone
C. extended radical ear surgery
D. tympanoplasty
E. shunting of the tympanic cavity

35. Which of the following is confirmed by the hydrops of the labyrinth in


Meniere's disease?
A. pneumatic breakdown
B. ewald breakdown
C. rinne experience
D. glycerol test
E. caloric breakdown

36. Which of the following preparations is more appropriate to use for stopping an
attack in Meniere's disease?
A. acetylsalicylic acid 1.0 inside
B. 0.1% atropine sulfate solution 1 ml into the muscle
C. 50% solution of analgin 2 ml into the muscle
D. 5% solution of ascorbic acid 2 ml into a vein
E. penicillin solution

37. A patient with purulent epitympanitis sharply worsened the general condition:
a diffuse intense headache, clouding of consciousness, body temperature rose to 40
° C. The examination revealed meningeal signs. Cerebrospinal fluid showed high
pleocytosis due to neutrophils. Otogenic purulent meningitis was diagnosed.
What medical tactics should be applied in this particular case?
A. lumbar puncture and antibiotic therapy
B. sanitizing ear surgery and intravenous antibiotic therapy
C. sanitizing ear surgery and ultraviolet blood irradiation
D. sanitizing ear surgery and lumbar puncture
E. sanitizing ear surgery and intracarotid antibiotic therapy

38. The patient complains of coordination disorder, hearing loss and suppuration
from the right ear. On examination: subtotal defect of the right tympanic
membrane, purulent discharge with an unpleasant odor, and bone caries in the
tympanic cavity. Computed tomography revealed a cerebellar abscess. What
medical tactics should be used in this case with the patient?
A. sanitizing surgery on the right ear and opening of the cerebellar abscess
B. opening of the cerebellar abscess and lumbar puncture
C. sanitizing surgery on the right ear and decongestant therapy
D. sanitizing surgery on the right ear and antibiotic therapy
E. lancing of the cerebellar abscess and ultraviolet blood irradiation

39. The patient has been suffering from epitympanitis for many years. He
categorically refuses surgical treatment, he is treated independently with folk
methods. I went to the doctor in connection with the “skewed” face that appeared
during the last exacerbation. Examination revealed paresis of the left facial nerve.
Choose a plan of medical tactics in this particular case?
A. hospitalization in the neurological department, physiotherapy
B. hospitalization in the ENT department, antibiotic therapy
C. hospitalization in the neurological department, acupuncture
D. admission to the neurological department, massage
E. hospitalization in the ENT department, sanitizing ear surgery

40. The unconscious patient was taken to the hospital. Temperature 38.7 ° C,
screaming in pain. According to his wife, for 2 years suppuration from the right
ear practically did not stop, a week ago he began to complain of a headache, 2
hours ago his consciousness became confused. Computed tomography revealed an
abscess in the temporal lobe of the brain.
Decide to which department should the patient be hospitalized?
A. department of efferent methods
B. neurological department
C. intensive care unit
D. neurosurgical department
E. ENT department

41. What method is most informative in the diagnosis of brain abscesses?


A. x-ray of the skull
B. angiography of the brain
C. echoencephalography
D. computed tomography of the brain
E. electroencephalography

42. In a patient delivered urgently during lumbar puncture, meningeal signs were
revealed: pleocytosis due to neutrophils in the cerebrospinal fluid. According to
relatives, it is known that the condition worsened dramatically 3 days ago, after
suffering from acute respiratory viral infection. Since childhood, suffers from
chronic purulent epitympanitis. Otogenic purulent meningitis was diagnosed.
Choose which method of treatment is preferable for the patient?
A. lumbar puncture
B. sanitizing ear surgery
C. ultraviolet blood irradiation
D. intravenous antibiotic therapy
E. intracarotid antibiotic therapy
43. The patient complains of hearing loss in both ears within 2 weeks, after
suffering from acute respiratory viral infections, noise and ringing in the ears. No
pain in the ears. Objectively: otoscopically, the eardrums are gray, slightly
retracted, the identification marks of the tympanic membrane are clearly expressed.
Select which study should be performed to objectively assess the condition of the
patient's auditory analyzer?
A. study of the patency of the Eustachian tube
B. external examination
C.otoscopy
D. mastoid radiography
E. computer audiometry
44. A patient who was admitted to the hospital on an emergency basis with a
temperature increased to 38.3 ° C, from childhood notes periodic suppuration from
the ear. With the last exacerbation, a diffuse headache, nausea and vomiting, not
associated with food intake, appeared. The stiffness of the muscles of the occiput is
determined, Kernig's symptom. Otogenic meningitis is suspected. Select what
additional research is needed to clarify the diagnosis and predict treatment tactics
in a patient?
A. CT scan of the head
B. ear discharge
C. lumbar puncture
E. x-ray of the skull
E. complete blood count

45. A patient who has been observed for a long time for epitympanitis (refused
surgical treatment), with the next exacerbation, developed a headache in the
temporal region, low-grade fever. A neuropathologist revealed focal symptoms, an
ophthalmologist - stagnation of veins in the fundus. On CT of the skull, a focus in
the temporal lobe of the brain is determined. Diagnosed with an otogenic brain
abscess. Evaluate the patient's treatment algorithm in this situation?
A. sanitizing ear surgery and antibiotic therapy
B. sanitizing ear surgery and lumbar puncture
C. lumbar punctures and antibiotic therapy
D. sanitizing ear surgery and lancing of the brain abscess
E. lancing of the brain abscess and antibiotic therapy

46. A 8-year-old child after suffering otogenic meningitis developed IV degree


hearing loss. What are the rehabilitation measures for this child?
A. studying in a general education school, using a hearing aid
B. training in a specialized school, using a hearing aid
C. teaching in a comprehensive school, the first desk
E. homeschooling, classes with a deaf teacher
E. education in a comprehensive school, classes with a deaf teacher

47. Specify the name of the operation for chronic suppurative otitis media,
aggravated by meningitis:
A. antromastoidotomy
B. radical surgery on the temporal bone
C. extended radical ear surgery
D. tympanoplasty
E. shunting of the tympanic cavity.

48. Specify the main instrumental and invasive methods for diagnosing an otogenic
abscess of the temporal lobe of the brain:
A. spinal puncture
B. rheoencephalography
C. m-echoscopy, electroencephalography
D. carotid angiography, computed tomography, M - echoscopy,
electroencephalography
E. x-ray of the skull.

49. A patient with purulent epitympanitis sharply worsened the general condition:
a diffuse intense headache, clouding of consciousness, body temperature rose to 40
° C. The examination revealed meningeal signs. Cerebrospinal fluid showed high
pleocytosis due to neutrophils. Otogenic purulent meningitis was diagnosed.
What medical tactics should be applied in this particular case?
A. lumbar puncture and antibiotic therapy
B. sanitizing ear surgery and intravenous antibiotic therapy
C. sanitizing ear surgery and ultraviolet blood irradiation
D. sanitizing ear surgery and lumbar puncture
E. sanitizing ear surgery and intracarotid antibiotic therapy

50. The patient complains of coordination disorder, hearing loss and suppuration
from the right ear. On examination: subtotal defect of the right tympanic
membrane, purulent discharge with an unpleasant odor, and bone caries in the
tympanic cavity. Computed tomography revealed a cerebellar abscess. What
medical tactics should be used in this case with the patient?
A. sanitizing surgery on the right ear and opening of the cerebellar abscess
B. opening of the cerebellar abscess and lumbar puncture
C. sanitizing surgery on the right ear and decongestant therapy
D. sanitizing surgery on the right ear and antibiotic therapy
E. lancing of the cerebellar abscess and ultraviolet blood irradiation

51. The patient has been suffering from epitympanitis for many years. He
categorically refuses surgical treatment, he is treated independently with folk
methods. I went to the doctor in connection with the “skewed” face that appeared
during the last exacerbation. Examination revealed paresis of the left facial nerve.
Choose a plan of medical tactics in this particular case?
A. hospitalization in the neurological department, physiotherapy
B. hospitalization in the ENT department, antibiotic therapy
C. hospitalization in the neurological department, acupuncture
D. admission to the neurological department, massage
E. hospitalization in the ENT department, sanitizing ear surgery

52. The unconscious patient was taken to the hospital. Temperature 38.7 ° C,
screaming in pain. According to his wife, for 2 years suppuration from the right
ear practically did not stop, a week ago he began to complain of a headache, 2
hours ago his consciousness became confused. Computed tomography revealed an
abscess in the temporal lobe of the brain.
Decide to which department should the patient be hospitalized?
A. department of efferent methods
B. neurological department
C. intensive care unit
D. neurosurgical department
E. ENT department

53. The greatest diagnostic value in otogenic sepsis is:


A. bacteriological blood test
B. serological blood test
C. CSF study
D. urine examination
E. immunological study
54. Cloudy cerebrospinal fluid, cytosis up to 30,000 cells in 1 μl, is released from
the needle under a pressure of 500 mm of water. st. at:
A. brain abscess
B. arachnoiditis
C. meningitis
D. normal
E. sepsis
55. Preferable for purulent meningitis administration of antibiotics:
A. intramuscularly
B. subcutaneously
C. into the cubital vein
D. into the subclavian vein through a catheter
E. per os

56. A patient with chronic purulent inflammation of the middle ear developed an
asymmetry of the face in the form of lowering of the corner of the mouth,
flattening of the nasolabial fold, non-closure of the palpebral fissure on the
diseased side. What complication of chronic suppurative otitis media should be
predicted?
A. thrombophlebitis of the sigmoid sinus
B. otogenic arachnoiditis
C. togenic meningitis
D. facial nerve neuritis
E. labyrinthitis

57. The patient 44 years old complaints about burning sensation, sore throat. The


temperature is normal. In the area of the upper pole of the left palatine tonsil, there
is a superficial ulceration with uneven edges, covered with a grayish bloom,
bleeding easily when touched. On palpation around the ulcer, a woody density of
infiltration is determined, passing to the soft palate and the root of the tongue. On
the left neck, a conglomerate of enlarged lymph nodes is palpable, 5x6 cm in size,
dense, limited displacement. The purpose of what method and research is
appropriate to establish an accurate diagnosis?

 A. roentgenogram
B. biopsy
C. computer system CT scan
D. nuclear magnetic field resonance
E. ultrasound
 
 
58. A 5- year-old child with adenoid vegetation has severe difficulty breathing
through the nose. The child, according to the mother, breathes through his mouth,
sleeps restlessly at night, cries out in his sleep, often suffers from respiratory
diseases, and suffered pneumonia. The child is pale, the facial skull is extended in
a vertical direction. With posterior rhinoscopy, pink lobules are determined,
covering the choanas by 2/3. Which of the following treatments is most
appropriate?

 A. adenotomy
B. tonsillotomy
C. tonsillectomy
D. conservative treatment
E. physiotherapeutic procedure

 
59. A 4- year-old child does not breathe well through his nose, often suffers from
respiratory diseases, has had a sore throat several times, does not sleep well, cries
out in his sleep, bedwetting is noted. Objectively: pallor of the skin, half-open
mouth. On examination, the smoothness of the nasolabial folds. High Gothic
sky. Which of the following listed below the methods surveys are the most popular
appropriate?
 A. anterior rhinoscopy, radiograph nasopharynx
B. posterior rhinoscopy, radiograph nasopharynx
C. metavariscite, radiograph nasopharynx
D. ultrasound examination
E. x-ray of the appendages sinuses of the nose

60. A 25-year-old patient complains of a slight pain in the throat on the left. The


general condition is not disturbed. Temperature Z6.6 0 С. Changes in the internal
organs are not determined. At the upper pole of the left tonsil there is a grayish
plaque, after removal of which a rather deep ulcer with an uneven sebaceous
bottom was found. On the part of other ENT organs, deviations from the norm are
not determined. The presumptive diagnosis of syphilis is a chancre 1 tbsp. Which
of the following research methods is the most appropriate for clarifying the
diagnosis?
 
 
 A. general analysis blood
B. blood biochemistry
C. blood test on RW
D. blood test on HIV
E. blood test for hepatitis B

 
61. Patient 16 years old was delivered on a stretcher with complaints of severe
weakness, malaise, headache, pain in the throat. The second day is ill, about a
week ago he was in contact with a sick child who was admitted to the infectious
diseases department with a diagnosis of diphtheria. The patient's condition is
moderate, weakened. Temperature 38.6 ºС, pulse 82 per minute. The mucosa is
hyperemic tonsils, covered with dirty - gray patina that apply to the bow, strikes
are difficult to remove, subject to the tissue bleeds. In the submandibular region -
swelling of the soft tissues of the neck. Which of the following treatments is most
appropriate?

 A. bacteriological testing research


B. soft tissue biopsy fabrics
C. bacteriological testing research
D. General analysis blood
E. General analysis urine

62. A 14-year-old patient was admitted on a stretcher with complaints of severe


weakness, malaise, headache, pain in the throat. The second day is sick, about a
week ago he was in contact with a sick child who was admitted to the infectious
diseases department with a diagnosis of diphtheria. The patient's condition is
moderate, weakened. Temperature 38.6 ºС, pulse 82 per minute. The mucous
membrane of the tonsils is hyperemic, covered with dirty gray deposits that spread
to the arches, the deposits are removed with difficulty, the underlying tissue is
bleeding. In the submandibular region - swelling of the soft tissues of the
neck. Which etiotropic treatment from the following examples should be applied in
this situation?
 
 A. introduction anti-diphtheria treatment serums
B. assign saline-alkaline rinsing
C. assign expectorants funds
D. to produce infusion the larynx chymotrypsin
E. to appoint antihistamines drugs

63. A patient is 22 years old, complains of hard pain in the throat, temperature 38 °
C, weakness, lethargy. Sick on the 2nd day, after hypothermia. General condition
of moderate severity, t 38.10 C, pale skin, enlarged regional lymph nodes of the
pharynx are palpable. Pharyngoscopy: determination of hyperemia and infiltration
of the soft palate, palatine arches, edematous and hyperemic palatine tonsils, on the
surface of which there are numerous round, slightly raised yellow dots. Which of
the following treatments is most appropriate?
 
 A. desensitizing therapy
B. vitamin therapy
C. antimicrobial therapy
D. physiotherapy
E., antiviral drug

 
64. A 36 year old patient has complaints of hard pain in the throat when
swallowing, chills, temperature 39 ° C, weakness, joint pain,
putrid breath. Objectively: the general condition is severe, t 39.6 0 C, the skin is
grayish, enlarged regional lymph nodes of the pharynx are
palpable. Pharyngoscopy: determined hyperemia and infiltration of the soft palate
and palatine arches, increase and hyperemia of the tonsils, purulent - fibrinous
plaque on the surface of the tonsils. The general analysis of blood revealed a shift
of the leukogram to the left. Which of the following examination methods is
the most optimal?
 
 A. smear on the flora from the throat
B. general blood test
C. the swab for yeast cells
D. smear on a spindle sticks
E. general analysis blood

 
65. The patient 46 years, complaining of severe pain in the throat when
swallowing, fever, t 39 0 C, weakness, joint pain, putrid breath. Objectively: the
general condition is severe, t 39.6 0 C, the skin is grayish, enlarged regional lymph
nodes of the pharynx are palpable. Pharyngoscopy: determined hyperemia and
infiltration of the soft palate and palatine arches, increase and hyperemia of the
tonsils, purulent - fibrinous plaque on the surface of the tonsils. In the general
analysis of blood revealed a shift in the g leukogram to the left. Which of the
following methods of treatment is the most optimal?
 
 A. antimicrobial therapy
B. vitamin therapy
C. physiotherapy
D. general strengthening therapy
E. antiviral therapy

66. The patient 17 years old, diphtheria. complaining of severe pain in the throat,
chills, t 39 0 C, weakness, joint pain, headache. Throat hurts 3rd day. Objectively:
general condition of moderate severity, t 38.6 0 C, pale skin, enlarged regional
lymph nodes are palpable. Opening the mouth is free. Pharyngoscopy: the soft
palate is symmetrical, hyperemia and infiltration of the soft palate and arches,
enlarged and hyperemic palatine tonsils, covered with a dirty white coating. What
disease is it necessary to carry out a differential diagnosis with?
 
 A. catarrhal angina
B. ulcerative-necrotic disease angina
S. herpetic angina
D. scarlatina
E. monocytic angina, fungal disease defeat

 
67.  The patient 16 years of complaints of severe pain in the throat, chills, t 39 0 C,
weakness, pain in the joints. The throat hurts for 4 days, before that during the
week worried about weakness, lethargy, malaise, low-grade fever. Objectively: the
general condition is severe, t 39.6 0 C, the skin is grayish, enlarged cervical,
axillary, inguinal lymph nodes are palpable; the liver and spleen are
enlarged. Pharyngoscopy: determined hyperemia and infiltration of the soft palate
and palatine arches, enlarged and hyperemic palatine tonsils, covered with
fibrinous plaque. A general blood test revealed a large number of
atypical mononuclear cells. Which one is below listed treatment methods it is the
most probable?
 
 A. antimicrobial therapy
B. antimicrobial therapy
C. anti-inflammatory therapy
D. antimicrobial therapy
E. antiviral therapy
68. The patient 28 years, complaining of severe pain in the throat, more on the
right, t 38,2 0 C, weakness. The 5th day is  sick, he was treated
by gargling. Objectively: the voice is nasal, the skin is pale, temperature 38.6 ° C,
enlarged submandibular lymph nodes are palpable, sharply painful on the right.
Pharyngoscopy:  the mouth opens wide on one finger, asymmetry hyperemia of the
soft palate, due to infiltration and edema of the right front of the palatine arches,
tonsils enlarged and hyperemic in the gaps of the tonsils purulent congestion, the
right tonsil is shifted to the center, swelling of the tongue. Which one is
below listed treatment methods it is the most correct?
 
 A. antibioticotherapia
B. surgical treatment
C. anti-diphtheria serum
D. antibioticotherapia
E. antiviral therapy

69. Child 9 years old was taken to the emergency room by the


ambulance team with complaints of difficulty breathing, severe sore throat,
temperature 38.2 ° C, weakness. From the words of the parents, it is known that the
third day is sick. Objectively: a serious condition, stridor breathing, a hoarse voice,
pale skin, temperature 38.6 ° C, enlarged, sharply painful regional lymph nodes are
palpable, the head is tilted to the left. Pharyngoscopy asymmetry, hyperemia and
protrusion of the posterior pharyngeal wall, more on the left. When palpating with
a blunt probe, a soft swelling is determined. Which of the following methods of
treatment is the most optimal?
 A. opening of an abscess, antibiotic therapy
B. opening of an abscess, antibiotic therapy
C. antibioticotherapia
D. surgical treatment
E. antibioticotherapia

70. The patient 19 years has complaints of frequent tonsillitis, recurrent pain in the
joints, and increased fatigue. From the anamnesis it is known that for 3 years every
six months she has been sick with tonsillitis, a year ago she had
paratonsillitis. Objectively: the general condition is satisfactory, the skin is of
normal color, the lower eyelids are cyanotic, enlarged regional lymph nodes are
palpable, the body temperature is normal. Pharyngoscopy determined palatine
tonsils closing in the midline, purulent plugs in the lacunae. Which of the
following are the most likely diagnoses and treatments?
 
 A. chronic tonsillitis. Double-sided tonsillectomy
B. paratonsillar abscess, autopsy an abscess
C. retropharyngeal abscess, autopsy abscess
D. chronic tonsillitis II art. Double-sided tonsillectomy
E. peripharyngeal abscess, autopsy abscess
 
71. The patient 18 years of complaints about frequent angina (2-3 times a
year). Objectively: the general condition is satisfactory, the skin is of normal color,
the regional lymph nodes are not palpable. Pharyngoscopy: palatine tonsils behind
the arches, soldered to them, hyperemia of the edges of the palatine arches, roller-
like thickening and swelling of the edges of the anterior and posterior
palatine. When pressed with a spatula on the palatine tonsils, liquid pus is released
from the lacunae. Which of the following are the most likely diagnoses
and treatments?
 
 A. chronic tonsillitis. Double-sided tonsillectomy
B. paratonsillar abscess, autopsy an abscess
C. retropharyngeal abscess, autopsy abscess
D. chronic tonsillitis II article, two-Sided tonsillectomy
E. peripharyngeal abscess, autopsy abscess

72. Patient 5 4 years of complaints of dryness and sore throat for a long


time. Objectively: the general condition is satisfactory, the skin is of normal color,
painless regional lymph nodes are palpable. Pharyngoscopy: the back wall throat
pale pink, dry with lacquer sheen. Which of the following are the most
likely diagnoses and treatments?
 
 A. atrophic pharyngitis immunomodulating, regenerating system therapy
B. acute tonsillitis, antimicrobial, anti-inflammatory therapy
C. acute pharyngitis, local antiseptic treatment
D. pharyngomycosis, local antifungal treatment
E. hypertrophic pharyngitis, quenching lapis

73. The patient 47 years has complaints of perspiration and dryness in the


throat. These complaints are disturbing for 6-8 months. With mesopharyngoscopy:
the mucous membrane of the posterior pharyngeal wall is hyperemic, thickened,
injected with vessels, covered with thick mucus; visible single
granules lymphadenoidnoy tissue. Which of the following are the most
likely diagnoses and treatments?
 
 A. atrophic pharyngitis immunomodulating, regenerating system therapy
B. acute tonsillitis, antimicrobial treatment
C. acute pharyngitis, local antiseptic treatment
D. pharyngomycosis, local antifungal treatment
E. hypertrophic pharyngitis, quenching lapis

74. The patient 47 years complaining of severe pain in the throat, worse


swallowing, on fever, malaise, weakness, headache. Ill for 3 days, the onset of the
disease is associated with hypothermia. Temperature 38.3 0 С, pulse - 88 bpm,
rhythmic. On palpation, the retromandibular and submandibular lymph nodes
are enlarged and painful. With mesopharyngoscopy bright hyperemia and swelling
of the mucous membrane of the tonsils, on the surface of which there are multiple
white dots the size of a millet grain. Which of the following are the most
likely diagnoses and treatments?
 
 A. lacunar angina, antibacterial angina
B. catarrhal angina, hyposensitizing therapy
C. follicular angina, antibacterial therapy
D. mixed angina, antibacterial angina
E. herpetic angina, irrigation throat antiseptics

75. Child 6 years old often suffers from respiratory diseases, 1 time in the last three
years has had a sore throat. There is a sharp increase in the palatine tonsils, which
are almost closed along the midline. There are no deposits and plugs in the
tonsils. There are no signs of chronic tonsillitis Which of the following diagnoses
and treatments is most likely?

A. chronic tonsillitis tonsillectomy


B. hypertrophy of the tonsils III St., tonsillotomy
C. hypertrophy of the tonsils III art., tonsillectomy
D. hypertrophy of the tonsils III ct., antibiotic therapy
E. chronic tonsillitis antibiotic therapy

76. The patient 47 years during the prophylactic examination, dense whitish-


yellow "plugs" were found on the surface of the palatine and lingual
tonsils. "Plugs" have a conical shape, rise above the surface of the mucous
membrane, are not removable, of dense consistency (bone). Which of the
following are the most likely diagnoses and treatments?

A. follicular angina; antibiotic therapy, local rinses throats


B. leptotrichia throat; antibiotic therapy, local rinsing throat
C. leptotrichia throat; Tubus-quartz, vitamin - and allergen therapy
D. lacunar sore throat; quartz tube, vitamin and mineral supplements
hyposensitizing therapy
E. ulcerative-necrotic disease sore throat; quartz tube, vitamin and mineral
supplements hyposensitizing therapy

77. A 23- year-old patient during the medical examination revealed a grayish-


yellow plaque on the surface of the right palatine tonsil. After removing the plaque,
an ulcer with uneven edges and a bleeding surface is visible. Body temperature
37.1 C, enlarged submandibular lymph nodes on the right. Which of the following
are the most likely preliminary diagnoses? What is necessary to study e to confirm
the diagnosis?
 A. ulcerative-necrotic disease angina; detection in a smear of yeast-like cells
B. ulcerative-necrotic angina; detection in a smear of pale Treponema
C. pharyngomycosis, detection in a smear of yeast-like cells
D. ulcerative-necrotic disease angina; detection in the fusiform smear sticks
and buccal spirochetes
E. diphtheria of the pharynx, detection in a smear of diphtheria stick

78. A 11 year old girl has complaints of perspiration, a burning sensation in her


throat, which appeared after she ate ice cream yesterday. Determined by hyperemia
of the mucous membrane of the posterior pharyngeal wall and its moderate
infiltration. On the part of other ENT organs, no noticeable changes are
detected. Which of the following are the most likely diagnoses and treatments?
 A. strep throat; a gentle diet, rinsing, symptomatic treatment treatment
B. acute tonsillitis; antibiotic therapy, symptomatic treatment treatment
C. strep throat; antibiotic therapy, symptomatic treatment treatment
D. acute tonsillitis; a gentle diet, rinsing, symptomatic treatment treatment
E. acute tonsillitis; tonsillotomy, symptomatic treatment treatment

79. A 43-year-old patient has complaints of dryness and scratching in the throat,
expectoration of viscous sputum. Determined by local signs of chronic tonsillitis.
Which one is below listed treatment methods most likely?
 A. conservative treatment of chronic tonsillitis (flushing tonsillar diseases
gap)
B. surgical treatment treatment (tonsillotomy)
C. surgical treatment treatment (tonsillectomy)
D. symptomatic treatment treatment, vitamin therapy
E. physiotherapy, vitamin therapy

80. Patient 65 years old, complains of dryness and scratching in the throat,


sometimes pain when swallowing solid food, bad breath. Sick for more than 5
years, suffers from chronic achilic gastritis and cholecystitis. The mucous
membrane of the pharynx is pale, thinned, dry, covered with viscous
mucus. Which of the following are the most likely diagnoses and treatments?
 A. atrophic pharyngitis. Iodide drugs, Vit A, E, B, rinse aid
B. atrophic pharyngitis. Antibiotics, rinses
C. hypertrophic pharyngitis. Antibiotics, rinses
D. hypertrophic pharyngitis. Braising lapis lazuli, gargle
E. the faringomikoza. Lubrication the mucosa mycotic agents

81. Patient 19 years of complaints of severe pain in the throat, worse swallowing, on


fever, malaise, headache. She fell ill 3 days ago after hypothermia. Temperature 38.3
C, pulse - 88 per minute, rhythmic. Determined by hyperemia of the mucous
membrane of the tonsils. On the surface of the tonsils, white dots the size of a millet
grain. Submandibular lymph nodes are enlarged, painful on palpation. Which of the
following are the most likely diagnoses and treatments?
 A. mixed angina, antibacterial drugs funds
B. herpetic angina, antiviral drugs funds
C. ulcerative-necrotic disease angina, antibacterial drugs funds
D. catarrhal sore throat, local treatment with antiseptics
E. follicular angina, antibacterial drugs funds

82. A 40-year-old patient complains of a pain in the throat at rest and during


swallowing, fever, general malaise, weakness. The skin is moist, the temperature is
38.6 C. And there is hyperemia of the mucous membrane of the tonsils, white
plaque emanating from the lacunae. Plaques are easily removed, do not spread
beyond the surface of the tonsils. Submandibular and cervical lymph nodes are
enlarged, indurated, painful on palpation. Which of the following are the most
likely diagnoses and treatments?
 A. lacunar angina, antibacterial drugs funds
B. herpetic angina, antiviral drugs funds
C. ulcerative-necrotic disease angina, antibacterial drugs funds
D. catarrhal sore throat, local treatment with antiseptics
E. follicular angina, antibacterial drugs funds

83. A 7- year-old child complains of pain in the throat, hunger pain, temperature up


to 39.2 C, vomiting, the mucous membrane of the pharynx is hyperemic, on the
palatine tonsils, palatine arches, on the mucous membrane of the cheeks there are
pinkish-white bubbles. The submandibular lymph nodes are enlarged and painful,
in the blood - leukocytosis - 10. Which of the following are the most likely
diagnoses and treatments?

A. lacunar angina, antibacterial drugs funds


B. herpetic angina, antiviral drugs
C. ulcerative-necrotic disease angina, antibacterial drugs funds
D. catarrhal sore throat, local treatment with antiseptics
E. follicular angina, antibacterial drugs funds

84. A 36-year-old patient has complaints of pain in the throat when swallowing,
fever up to 38.8 ° C, general malaise. Ill for 3 days. She was treated with home
remedies - gargling, taking aspirin, the treatment did not help. Pharyngoscopy is
difficult - it reacts painfully to the introduction of a spatula. The mucous
membrane of the pharynx is hyperemic, infiltrated. Submandibular lymph nodes
are enlarged, painful on palpation. With hypopharyngoscopy, hyperemia,
infiltration of the tissue of the lingual tonsil is determined, plaques in the form of
white islets are determined on its surface. Which of the following are the most
likely diagnoses and treatments?
 A. follicular angina, antibacterial drugs drugs
B. angina of the tongue tonsils, antibacterial drugs
C. lacunar angina, antibacterial drugs drugs
D. mixed angina, antibacterial drugs drugs
E. pharyngeal surgery angina, antibacterial drugs drugs

85. A 9- year-old child has difficulty in nasal breathing, mucopurulent nasal


discharge, dry throat, subfebrile condition. Ill for about 2 weeks. The mucous
membrane of the nose is pink, edematous in the region of the inferior turbinates,
mucous discharge in the region of the bottom of the nose on both sides. Pharyngeal
tonsil II degree of enlargement, edematous, hyperemic, on its surface there are
white plaques in the form of islands. The mucous membrane of the posterior
pharyngeal wall is hyperemic, covered with thick secretions. Which of the
following are the most likely diagnoses and treatments?
 A. adenoid cells vegetation conditions II, purulent adenoiditis. Antibacterial
preparation
B. adenoid cells growing season W, purulent adenoiditis. Antibacterial preparation
C. adenoid cells vegetation period I, purulent adenoiditis. Antibacterial preparation
D. pharyngeal surgery angina, antibacterial drugs drugs
E. catarrhal angina, antibacterial drugs drugs

86. The patient 28 years of complaints of pain in the throat, worse swallowing,


more on the right, pain in the right ear, difficulty in swallowing, and when you
open your mouth, malaise, sick 5 days to her back after transferred catarrhal
angina. Mouth opens to one cross section the finger.  In the pharynx, hyperemia of
the mucous membrane is determined, infiltration about the tnsillitis region on the
right, the right tonsil is shifted to the midline. The uvula is sharply swollen and
displaced somewhat to the left. Submandibular lymphonoduses on the right
are indurated, enlarged and painful on palpation. Which of the following are the
most likely diagnoses and treatments?
 A. paratonsillar abscess on the left. Opening of the abscess
B. retropharyngeal abscess. Autopsy an abscess
C. peritonsillar abscess of the right. Opening of the abscess
D. peripharyngeal abscess. Autopsy an abscess
E. ulcerative-necrotic disease angina. Antibacterial agen

87. A 12-year-old boy has complaints of annual angina with fever up to 38-39-C
and general disturbance.  With mesopharyngoscopy: injection of vessels along the
edge of the palatine arches, tonsils are scarred, dense, protrude beyond the valve of
the arches, with purulent plugs in the lacunae. The
submandibular retromandibular and upper cervical lymph nodes are enlarged and
painless. Which of the following are the most likely diagnoses and treatments?
 A. chronic tonsillitis simple form. Conservative treatment
B. paratonsillar abscess, autopsy an abscess
C. chronic tonsillitis II art. Double-sided tonsillectomy
D. peripharyngeal abscess, autopsy abscess
E. retropharyngeal abscess, autopsy abscess

88. A 32-year-old patient consulted an ENT doctor with complaints of bursting


pain in the nose and difficulty in nasal breathing. According to the patient - 3 days
ago he received a blow in the area of the nose. With anterior rhinoscopy, the nasal
passages are sharply narrowed due to inflammatory edema of the mucous
membrane in the anterior sections of the nasal septum on both sides, nasal
breathing is difficult. When probing, there is a symptom of fluctuation, pain.
Choose the most appropriate method of treatment for this patient?

A. vasotomy of the nose


B. conchotomy
C. lancing of an abscess
D. polypotomy
E. electrocaustics

89. In a patient delivered urgently during lumbar puncture, meningeal signs were
revealed: pleocytosis due to neutrophils in the cerebrospinal fluid. According to
relatives, it is known that the condition worsened dramatically 3 days ago, after
suffering from acute respiratory viral infection. Since childhood, suffers from
chronic purulent epitympanitis. Otogenic purulent meningitis was diagnosed.
Choose which method of treatment is preferable for the patient?
A. lumbar puncture
B. sanitizing ear surgery
C. ultraviolet blood irradiation
D. intravenous antibiotic therapy
E. intracarotid antibiotic therapy
90. The patient complains of hearing loss in both ears within 2 weeks, after
suffering from acute respiratory viral infections, noise and ringing in the ears. No
pain in the ears. Objectively: otoscopically, the eardrums are gray, slightly
retracted, the identification marks of the tympanic membrane are clearly expressed.
Select which study should be performed to objectively assess the condition of the
patient's auditory analyzer?
A. study of the patency of the Eustachian tube
B. external examination
C. otoscopy
D. mastoid radiography
E. computer audiometry

91. A patient who was admitted to the hospital on an emergency basis with a
temperature increased to 38, 3 ° C, from childhood notes periodic suppuration from
the ear. With the last exacerbation, a diffuse headache, nausea and vomiting, not
associated with food intake, appeared. The stiffness of the muscles of the occiput is
determined, Kernig's symptom. Otogenic meningitis is suspected. Select what
additional research is needed to clarify the diagnosis and predict treatment tactics
in a patient?
A. CT scan of the head
B. ear discharge
C. lumbar puncture
E. x-ray of the skull
E. complete blood count
92. A patient who has been observed for a long time for epitympanitis (refused
surgical treatment), with the next exacerbation, developed a headache in the
temporal region, low-grade fever. A neuropathologist revealed focal symptoms, an
ophthalmologist - stagnation of veins in the fundus. On CT of the skull, a focus in
the temporal lobe of the brain is determined. Diagnosed with an otogenic brain
abscess. Evaluate the patient's treatment algorithm in this situation?
A. sanitizing ear surgery and antibiotic therapy
B. sanitizing ear surgery and lumbar puncture
C. lumbar punctures and antibiotic therapy
E. sanitizing ear surgery and lancing of the brain abscess
E. lancing of the brain abscess and antibiotic therapy

93. Patient 30 years old, complained of shortness of breath, hoarseness, noisy


breathing. According to the patient, the above complaints appeared after eating
honey. Objectively: the mucous membrane of oropharynx is edematous, the uvula -
vitreous edema. With indirect laryngoscopy, pronounced edema of the mucous
membrane is determined, the glottis is 2-3 mm. Diagnosed with allergic laryngeal
edema. Stage III stenosis. What is the treatment for laryngeal stenosis stage III of
allergic genesis?
A. tracheostomy
B. conservative
C. conicotomy
D. resection of the larynx
E. tracheotomy
94. The patient was admitted to the ENT department with complaints of difficulty
breathing through the natural airways. From the anamnesis: considers himself sick
within 2 hours after eating honey, called the ambulance service, delivered on
emergency indications. Objectively: vitreous edema of the laryngeal mucosa is
determined. The voice is husky. The glottis is 6-7 mm. Diagnosed with allergic
laryngeal edema. Stage I stenosis. Use the drug most suitable for providing a
pronounced anti-inflammatory effect.
A. antibiotics
B. antihistamines
C. non-steroidal anti-inflammatory drugs
D. corticosteroids
E. decongestants
95. A 56-year-old patient suffering from hypertension was taken to the ENT clinic
due to profuse nosebleeds that could not be stopped at home. Objectively: BP
160/100 mm Hg, PS 80 per minute, with anterior rhinoscopy in the right half of the
nose, in the cartilaginous part of the nasal septum, the source of bleeding is
determined. A preliminary diagnosis was made: Recurrent epistaxis. Hypertonic
disease. What urgent measures need to be taken?
A. tamponade the nasal cavity
B. refer to a hematologist
C. refer to a therapist
D. X-ray of the paranasal sinuses
E. perform lumbar puncture
96. A bee got into the man's right ear while walking through the forest. The patient
feels severe pain in the ear, burning sensation, hearing loss. Objectively: the skin
of the auricle and the external auditory canal is sharply thickened, hyperemic. Part
of the insect's body is visible in the depths of the external auditory canal. A
preliminary diagnosis was made: Foreign body of the external auditory canal AD.
Which of the following methods is most appropriate for removing a foreign body?
A. removal by flushing
B. removal with tweezers
C. removal with a wire loop
D. removal with a curved hook
E. removal with alligator forceps
97. A 56-year-old patient, suffering from hypertension, consulted an ENT doctor
about profuse nosebleeds that could not be stopped at home. Objectively: BP
160/100 mm Hg, PS 80 per minute, with anterior rhinoscopy in the right half of the
nose, in the cartilaginous part of the nasal septum, the source of bleeding is
determined. A preliminary diagnosis was made: Recurrent epistaxis. Hypertonic
disease. Choose the most appropriate method for stopping nosebleeds on an
outpatient basis.
A. hydraulic separation
B. anterior tamponade
C. posterior tamponade
D. balloon tamponade
E. selective vascular embolization
98. A 5-year-old child was taken to a doctor due to the fact that three days ago,
while playing, he pushed a pea into the ear canal. Complains of a feeling of
pressure in the ear, hearing loss. Objectively: the skin of the auricle is not changed,
a foreign body with a smooth surface was found in the external auditory canal,
obstructing the external auditory canal. Use the most correct method for removing
foreign bodies from the treated objects:
A. add alcohol and remove the foreign body with a crochet
B. wash with the solution using Janet's syringe
C. remove foreign body with tweezers
D. remove by loop
E. remove by crochet
99. A 15-year-old patient complains of pain in the nose, deformity of the nose.
According to the patient, he was injured in training. Also noted nosebleeds, which
stopped on their own. On examination: the shape of the nose is changed due to the
retraction of the left slope of the nose, crepitus is noted on palpation of the nasal
bones, and subcutaneous emphysema is also determined. Which of the following
research methods should be prescribed to the patient to clarify the diagnosis?
A. multispiral CT of the skull;
B. X-ray of the skull in two projections
C. diaphanoscopy
D. endoscopy of the nasal cavity
E. rhinopneumometry

100. A 68-year-old patient suffering from essential hypertension was admitted to


the ENT department due to nosebleeds. With anterior rhinoscopy, the source of
bleeding is immense. The doctor on duty performed an anterior tamponade of the
nasal cavity, however, with oropharyngoscopy, fresh blood is determined along the
posterior pharyngeal wall. Choose an additional method from those suggested for
stopping nosebleeds.
A. ligation of the external carotid artery from the side of bleeding
B. clipping a. Ethmoidalis
C. electroplating of a bleeding vessel
D. posterior nasal tamponade
E. embolization a. maxillaris

101. A 3-year-old child was brought to the ENT office with the parents' complaints
of difficulty in nasal breathing, with profuse purulent discharge from the left side
of the nose and pain for 3 weeks. Objectively: The general condition is
satisfactory, the temperature is normal, the skin of the left wing of the nose is
hyperemic, macerated, covered with dry crusts. In the left half of the nasal cavity
there is abundant mucopurulent discharge, the mucous membrane is hyperemic,
edematous. In the left half of the nose - a formation of a round shape, 1 cm in
diameter. A foreign body of the left half of the nose was diagnosed. Use the correct
method to remove the foreign body from those suggested.
A. using bayonet tweezers
B. with a padded jacket
C. with a wire hook
D. by washing
E. with a clamp

102. A 43-year-old man complained of a sudden onset of hearing loss and a sharp
pain in the right ear, which appeared after water getting into the ear while taking a
shower. Objectively: when pressing on the tragus, pain is noted, the skin of the
external auditory canal is sharply hyperemic, edematous. The external auditory
canal was obturated with a sulfur plug. The diagnosis was made: Sulfur plug of the
external auditory canal on the right. External diffuse otitis on the right. What
measures should be provided to the patient in order to relieve pain syndrome?
A. flush the external auditory canal
B. remove foreign body with tweezers
C. prescribe pain medications
D. alcohol-containing drops into the external auditory canal
E. remove the foreign body with a loop

103. Patient K., 35 years old, stung by a wasp on the cheek, 3-4 minutes later.
hyperemia and swelling appeared in this area, and then the limbs and the whole
body, accompanied by sharp itching. After half an hour, there was a growing
shortness of breath and difficulty in swallowing. Diagnosed with allergic laryngeal
edema. Select an emergency drug group.
A. glucocorticosteroids
B. decongestants
C. non-steroidal anti-inflammatory drugs
D. antibiotics
E. antihistamines

104. A 18-year-old patient presented with epistaxis. Such bleeding, more or less
profuse, occurs often in the premenstrual period. There is no bleeding at the time
of examination. The skin and visible mucous membranes are pale. The pulse is
rhythmic - 88 beats / min. Bloody clots in the nasal passages. On the left, in the
anterio-inferior part of the nasal septum, the vessels are sharply expanded. What is
the most appropriate way to deal with the patient in this case?
A. perform submucosal resection of the nasal septum
B. to carry out chemical or thermal cauterization of the vessels of the
Kisselbach zone
C. perform a vasotomy of the inferior turbinates
D. perform anterior tamponade of the nasal cavity
E. no treatment is prescribed
105. A 62-year-old patient complains of severe stabbing pain in the throat on the
left, painful swallowing. These symptoms appeared after eating fish an hour ago.
Objectively: removable dentures of the upper and lower jaw. The mucous
membrane of the left palatine tonsil is hyperemic. Fish bone is visible in the tissue
of the right palatine tonsil. Diagnosed with Foreign body of the oropharynx.
Choose from the offered options further treatment tactics.
A. tonsillectomy
B. adenotonsillotomy
C. removal of foreign body
D. prescribing anti-inflammatory treatment
E. prescribing antibacterial treatment

You might also like